You are on page 1of 100

Nhóm file Word toán THCS

123 BÀI HÌNH CHỌN LỌC ÔN THI VÀO LỚP 10

Bài 1. Cho nửa đường tròn tâm đường kính . Lấy một điểm trên nửa đường tròn sao cho
.Gọi là giao điểm của tiếp tuyến tại với nửa đường tròn và đường thẳng .

a) Chứng minh rằng .

b) Từ vẽ tiếp tuyến thứ hai với đường tròn tại , cắt tại . Tính , , theo .

c) Vẽ vuông góc với tại . Gọi là giao điểm của và . Tính theo .

Hướng dẫn giải

a) P
M
(góc nội tiếp chắn nửa đường tròn) C
Áp dụng hệ thức lương trong tam giác vuông tại ,
đường cao có . N I
b) Tính , , theo :

vuông tại , ta có
A O H B

.
Do (tính chất hai tiếp tuyến cắt nhau) và nên là trung trực của .

mà . Vậy .

c) Tính theo .

Ta có // (cùng vuông góc với ) nên và đồng dạng (g.g)

Suy ra .

Vậy , theo định lý Ta-let đảo ta được // .

Tam giác có (cmt); // (cmt) suy ra .

1
Nhóm file Word toán THCS

Vậy là đường trung bình của tam giác suy ra .

Lại có: .

Bài 2. Cho nửa đường tròn tâm đường kính . Gọi là trung điểm của đoạn thẳng , tia
vuông góc với đường thẳng , cắt nửa đường tròn tâm tại . Lấy là một điểm bất kỳ nằm trên đoạn
thẳng ( khác và ), tia cắt nửa đường tròn đã cho tại . Tiếp tuyến với nửa đường tròn tâm
tại cắt tại . Tia cắt tại .

a) Chứng minh bốn điểm , , , cùng thuộc một đường tròn.

b) Chứng minh tam giác cân.

c) Tính diện tích tam giác theo khi là trung điểm của đoạn thẳng .

d) Chứng minh rằng: Khi di động trên đoạn thẳng thì tâm của đường tròn ngoại tiếp tam giác nằm
trên một đường thẳng cố định.

Hướng dẫn giải

a) Chứng minh bốn điểm , , , cùng thuộc một đường tròn. D


(góc nội tiếp chắn nửa đường tròn).
Vậy và cùng nhìn dưới một góc vuông nên bốn điểm N
M
, , , cùng thuộc một đường tròn đường kính . I
b) Chứng minh tam giác cân.
Ta có nên tứ giác nội tiếp .
E A C O B
Mà (góc nội tiếp và góc tạo bởi tiếp tuyến và dây cùng

chắn ) cân tại .

c) Tính

(góc nội tiếp chắn nửa đường tròn)

có và , .

đồng dạng với tam giác (g.g) .

d) Tâm đường tròn ngoại tiếp tam giác nằm trên một đường tròn cố định
2
Nhóm file Word toán THCS
Dựng thuộc tia đối của tia sao cho cố định.

Ta có , vuông góc với tại nên cân tại


, mà (cùng phụ ) .

tứ giác nội tiếp đường tròn có tâm nằm trên trung trực của .

3
Nhóm file Word toán THCS
Bài 3. Gọi là điểm chính giữa cung của nửa đường tròn tâm đường kính , là điểm bất kỳ trên
cung . Kẻ vuông góc với tại , gọi là giao điểm của với , cắt đường tròn
tại .

a) Chứng minh rằng: Tam giác vuông cân và // .

b) Xác định vị trí của để , , thẳng hàng.c) Gọi là giao điểm của với . Chứng minh rằng:
vuông góc với .

Hướng dẫn giải

a) Chứng minh tam giác vuông cân và // .

Do là điểm chính giữa của nửa đường tròn nên E

, mà (gt)
tam giác vuông cân tại .
Ta có và (tam giác vuông cân),
suy ra là trung trực của . Vì thuộc nên C
suy ra tam giác cân tại . D
I M
Suy ra (hai góc nội tiếp cùng chắn cung ), mà
H
hai góc , ở vị trí so le trong nên // .
A O B

b) Định vị trí của điểm để , , thẳng hàng


N
, , thẳng hàng . Vậy
sao cho .
Cách dựng điểm :
Trên tia đối tia dựng điểm sao cho , cắt đường
tròn tại suy ra là điểm cần dựng.
Thật vậy, với cách dựng trên ta có: Tam giác đồng dạng với tam giác
(g.g) C
D
I
M
.
c) Chứng minh: vuông góc với . H
Ta có // (vì cùng vuông góc với )
A O B
Suy ra (so le trong). Lại có (hai góc nội tiếp
chắn cung ) (tính chất đối xứng) , mà
hai góc ở vị trí so le trong nên // .
Có ( ; góc nội tiếp chắn nửa đường tròn) .

4
Nhóm file Word toán THCS

Mặt khác // (cmt), ( ) suy ra .

Vậy là trực tâm của tam giác . Do đó .

Bài 4. Cho đường tròn đường kính . Gọi là điểm trên đường tròn . Tia phân giác góc
cắt tại và cắt đường tròn tại .

a) Chứng minh rằng: và tính theo .

b) Gọi , là hình chiếu của trên , . Tứ giác có dạng đặc biệt nào? Giải thích.

c) Cho . Tính , theo .

d) Gọi là bán kính đường tròn nội tiếp tam giác . Chứng minh rằng: ( là diện tích tam
giác ).

Hướng dẫn giải

a) , tính theo . A
P
Do là phân giác của nên . 1 2 F
N
(hai góc nội tiếp bằng nhau nên cung chắn bằng nhau) I
(hai cung bằng nhau nên hai dây bằng nhau). E
C
B D
Mặt khác (góc nội tiếp chắn nửa đường tròn). O

vuông cân tại .


b) Tứ giác là hình gì?
Ta có , là phân giác của suy ra tứ giác M
là hình vuông (hình chữ nhật có một đường chéo là phân giác).
c) , theo .

(góc ở tâm và nội tiếp cùng chắn cung )

là cạnh tam giác đều nội tiếp

và đều .

Áp dụng tính chất phân giác

và .

d) Chứng minh .

5
Nhóm file Word toán THCS
Gọi là tâm đường tròn nội tiếp tam giác , đường tròn này tiếp xúc với , lần lượt tại ,

là hình vuông .

Ta có .

Bài 5. Cho đường tròn và hai đường kính , bất kỳ. Tiếp tuyến tại của đường tròn cắt cá
đường thẳng và lần lượt tại và . Gọi , lần lượt là trung điểm của các đoạn thẳng và
.

a) Chứng minh tứ giác nội tiếp.

b) Chứng minh rằng: và .

c) Chứng minh trực tâm của tam giác là trung điểm của đoạn thẳng .

d) Hai đường kính và có trí như thế nào thì tam giác có diện tích nhỏ nhất? Tính diện tích nhỏ
nhất đó theo .

Hướng dẫn giải

a) Tứ giác nội tiếp B

(góc nội tiếp chắn nửa đường tròn)


(vì cùng phụ ). D
Tam giác có nên tam giác cân O I
C
tại .
H
Suy ra tứ giác nội tiếp (có góc
trong bằng góc ngoài tại đỉnh đối diện). E F
P A Q

b) và .
Áp dụng hệ thức lượng trong tam giác vuông có:

, và

Do đó .

Lại có: .

6
Nhóm file Word toán THCS

Vậy .

c) Chứng minh là trung điểm .

Kẻ vuông góc với ( ) và cắt tại là trực tâm của tam giác .

Ta có

Suy ra tam giác đồng dạng với tam giác ( , )

Mà (cùng phụ với ).

, mà hai góc ở vị trí đồng vị nên // .

Trong tam giác có // ; (gt) suy ra .

d) Tìm vị trí của , để đạt giá trị nhỏ nhất?

Ta có .
Áp dụng bất đẳng thức Cô-si với hai số dương, có

. Dấu “ ” xảy ra khi và chỉ khi vuông cân tại


vuông cân tại .

Vậy khi và vuông góc với nhau.

Bài 6. Cho tam giác ABC ( AB < AC) nội tiếp đường tròn (O;R) đường kính BC. Vẽ đường cao AH của tam
giác ABC. Đường tròn tâm K đường kính AH cắt AB, AC và đường tròn (O) lần lượt tại các điểm D, E và I.
a) Chứng minh tứ giác AEHD nội tiếp là hình chữ nhật và .
b) Chứng minh tứ giác BDEC nội tiếp được đường tròn và OA vuông góc với DE.
c) Chứng minh rằng nếu diện tích tam giác ABC gấp đôi diện tích hình chữ nhật AEHD thì tam giác ABC
vuông cân.
d) Chứng minh các đường thẳng AI, DE và BC đồng quy.
Hướng dẫn giải

7
Nhóm file Word toán THCS
a) AEHD là hình chữ nhật và
Ta có: ( góc nội tiếp chắn nửa đường tròn )

Tứ giác AEHD là hình chữ nhật.


ÁP dụng hệ thức lượng trong tam giác.
Tam giác ABH vuông tại H, đường cao HD có

b) Chứng minh tứ giác BDEC nội tiếp và


( hai góc nội tiếp cùng chắn trong (K))
( cùng phụ với )

Tứ giác BDEC nội tiếp (có góc trong bằng góc ngoài tại đỉnh đối diện).
Gọi S là giao điểm của OA và DE.

Ta có: ( có nên cân tại O)

(tứ giác BDEC nội tiếp)

( vuông tại A)

, nghĩa là .

c) thì vuông cân.

và có: chung, (cmt)


∽ (g.g)

, đường kính đường tròn (K))


(

Suy ra:

Ta lại có:

Đẳng thức xảy ra khi và chỉ khi


vuông cân tại A.
8
Nhóm file Word toán THCS
d) AI, DE, BC đồng quy
Do DE là đường kính của đường tròn (K) .
Gọi M là giao diểm của hai đường thẳng AI và BC.
Xét đường tròn (O) và (K) có AI là dây chung OK là trung trực của AI .
có hai đường cao AH, OK cắt nhau tại K nên K là trực tâm .
Mà (cmt)
M, K, D thẳng hàng (vì từ điểm K ngoài OA ta chỉ vẽ duy nhất một đường thẳng vuông góc với OA)
DE cũng qua M, nghĩa là ba đường thẳng AI, DE, BC đồng quy

Bài 7. Cho đường tròn đường kính , dây cung của vuông góc với tại sao cho
. Trên đoạn lấy điểm . Tia cắt đường tròn tại điểm thứ hai là .
a) Chứng minh tứ giác nội tiếp được trong đường tròn.
b) Chứng minh: .
c) Chứng minh: .
d) Xác định vị trí điểm sao cho chu vi tam giác đạt giá trị lớn nhất.
e) Giả sử là trung điểm . Xác định vị trí điểm của để đạt giá trị lớn nhất.
Hướng dẫn giải
a) Chứng minh tứ giác nội tiếp được trong đường tròn.
Ta có:
(góc nội tiếp chắn nửa đường tròn)

Suy ra tứ giác nội tiếp được trong đường tròn


b) Chứng minh:
Ta có: tại
(hai góc nội tiếp cùng chắn hai cung bằng nhau)

và có chung,
suy ra và đồng dạng theo ( g, g) ta có

c) Chứng minh:
Ta có: (góc nội tiếp chắn nửa đường tròn)
Áp dụng hệ thức lượng trong tam giác vuông có:
do đó
d) Định vị trí điểm sao cho chu vi tam giác đạt giá trị lớn nhất.

9
Nhóm file Word toán THCS

Đặt là chu vi tam giác có

Áp dụng bất đẳng thức , ta được

. Do đó

Dấu bằng xảy ra khi và chỉ khi


e) Định vị trí của để đạt giá trị lớn nhất

Ta có: .
nửa tam giác đều . Vậy .
Do là trung trực của .
có đều.
Dựng điểm trên sao cho , mà (hai góc nội tiếp cùng chắn )
đều.

Vậy:
( là đường kính).
Suy ra: max là điểm đối xứng của qua .

Bai 8. Cho đường tròn đường kính , lấy là điểm nằm giữa và . Vẽ dây vuông
góc với tại . Trên lấy điểm bất kỳ, tia cắt đường tròn tại ( ).
a) Chứng minh rằng tứ giác nội tiếp được đường tròn.
b) Xác định vị trí của để đạt giá trị nhỏ nhất.
c) Chứng minh rằng tiếp xúc với đường tròn ngoại tiếp tam giác .
d) Chứng minh rằng trọng tâm của tam giác thuộc một đường tròn cố định khi di động trên đoạn
thẳng .
Hướng dẫn giải

a)Tứ giác nội tiếp


(góc nội tiếp chắn nửa đường tròn).

Tứ giác nội tiếp đường tròn.


b) Định M để 2AM + AN đạt giá trị nhỏ nhất.

Ta có :
Áp dụng bất đẳng thức Cô-si cho hai số dương 2AM và AN , ta có:

10
Nhóm file Word toán THCS

(không đổi).

Dấu “=” xảy ra khi


c) AC tiếp xúc với đường tròn ngoại tiếp tam giác CMN.
Đường kính
Dựng đường tròn ngoại tiếp tam giác CMN, dựng tia tiếp tuyến Ax( nằm khác phía tia CA bờ CM)

Do đó AC là tiếp tuyến của đường tròn ngoại tiếp tam giác CMN.
d) Trâ tâm G của tam giác CAN thuộc một đường tròn cố định.
Gọi K là trung điểm của AN , I là trung điểm của OA.
(định lý đường kính và dây cung).

Do G là trọng tâm tam giác CAN nên .

Tam giác KAO vuông tại K, có KI là trung tuyến ứng với cạnh huyền nên

Dựng điểm J trên đoạn CI sao cho cố định.

Xét tam giác CIK có

Vậy G thuộc đường tròn tâm J bán kính .

Bài 9: Cho nửa đường tròn tâm O có đường kính AB=2R, Tiếp tuyến Ax, By với nửa đường tròn (Ax, By nằm
cùng nửa mặt phẳng so với bờ AB). Tiếp tuyến tại M với (O) (M khác A,B) cắt Ax, By lần lượt tại C,D.
a) Chứng minh tứ giác ACMO, OBDM nội tiếp được và AC+BD=CD.

b) Chứng minh: và AC.BD=R2.


c) Gọi N là giao điểm AD với BC, tia MN cắt AB tại H. Chứng minh N là trung điểm MH.

d) Cho , AB= 5cm. Tính: SAMB.

Hướng dẫn giải

11
Nhóm file Word toán THCS
a) Chứng minh tứ giác ACMO, OBDM nội tiếp được
y
và AC+BD=CD.
Ta có: Tứ giác
ACMO nội tiếp được đường tròn. D
Tương tự: Tứ giác OBDM nội tiếp được ( Vì tổng hai góc
x
đối bằng 1800).
Áp dụng tích chất hai tiếp tuyến cắt nhau ta có: K
AC=CM và BD=DM. M
CD=CM+DM=AC+BD.
b) Chứng minh: và AC.BD=R2. C

N
Ta có OC là phân giác , OD là phân giác
(Tính chất hai tiếp tuyến cắt nhau)
B
A H O
Mà và là hai góc kề bù .

có , : AC.BD=CM.DM=OM2=R2.
c) N là trung điểm MH.

Gọi K là giao điểm của tia BM và tia Ax

(Góc nội tiếp chắn nửa đường tròn).

(Góc tạo bởi tia tiếp tuyến và dây cùng chắn ).

cân tại C CM=CA mà (Cùng phụ với hai góc bằng nhau).

cân tại C CK=CM. Vậy CK=CA.

AC//BD (Vì cùng vuông góc với AB) (Hệ quả của định lí Ta-lét).

MN//BD//AC (Định lí Ta-lét đảo).

Do đó: mà AC=CK(cmt) NH=NM N là trung điểm MH.

Cách khác : AC//BD (Vì cùng vuông góc với AB) (Hệ quả của định lí Ta-lét).

MN//BD//AC.

Áp dụng hệ quả của định lí Ta-lét ta có: MN=NH


N là trung điểm MH.
12
Nhóm file Word toán THCS
d)Tính: SAMB.

và có: ; (Hai góc nội tiếp cùng chắn )

Vậy : .

Bài 10: Cho nửa đường tròn tâm O có đường kính AB=2R, Tiếp tuyến Ax, By với nửa đường tròn (Ax, By nằm
cùng nửa mặt phẳng so với bờ AB). Tiếp tuyến tại M thuộc (O) (M khác A,B) cắt Ax, By lần lượt tại C,D.

a) Chứng minh: AC.BD=R2 và .


b) OC cắt AM tại I, OD cắt MB tại J. Chứng minh: MIOJ là hình chữ nhật và tứ giác CIJD nội tiếp được
đường tròn.
c) Giả sử: AM=R Tính SABDC theo R.
d) Cho AM=3cm, MB=4cm. Tính bán kính đường tròn ngoại tiếp tứ giác CIJD.

Hướng dẫn giải

a) Chứng minh: AC.BD=R2 và . y

Áp dụng tích chất hai tiếp tuyến cắt nhau ta có: D


S

AC=CM và BD=DM và OC OD lần lượt là hai tia phân giác của hai góc kề x
N
bù , nên . M

Xét có , AC.BD=CM.DM=OM2=R2. C
K J
I
b)Chứng minh: MIOJ là hình chữ nhật và tứ giác CIJD nội tiếp được
đường tròn. B
A O

Ta có: AC=CM (cmt), OA=OM=R CO là đường trung trực của AM


.

Tương tự: .

13
Nhóm file Word toán THCS

(Góc nội tiếp chắn nửa đường tròn).


Tứ giác MIOJ là hình chữ nhật (Tứ giác có 3 góc vuông).

Gọi K là giao điểm của hai đường chéo MO và IJ KO=KJ cân tại K nên : . Do đó:
(Vì cùng phụ với hai góc bằng nhau).
Tứ giác CIJD nội tiếp được đường tròn. (Góc trong bằng góc ngoài của đỉnh đối diện).
c)Tính SABDC theo R.

OA=OM=AM=R đều

AC= OAtg = R.tg300 =


(Vì cùng phụ với )
BD = OBtg = R.tg600 = R

(đvdt)
d) Tính bán kính đường tròn ngoại tiếp tứ giác CIJD.
Gọi N là trung điểm của CD. Đường vuông góc với CD tại N và đường vuông góc với IJ tại K cắt nhau tại
S.
SK,SN là đường trung trực của IJ và CD S là tâm đường tròn ngoại tiếp tứ giác CIJD.

vuông tại A, AI OC ta có OA2 = OI.OC

Mà BA2 = AM2 + MB2 = 32 + 42 = 52 ;

Vậy :

COD và AMB có: ; (Cùng phụ với hai góc bằng nhau ).
(g.g)

(Tỉ số hai đường trung tuyến tương ứng bằng tỉ số đồng dạng).

CS2 = SN2 + CN2 = OK2 + ON2 ( , đường trung tuyến ứng với cạnh huyền)

14
Nhóm file Word toán THCS
(Do tứ giác OKSN có các cạnh đối đôi một sông sông với nha nên OKSN là hình bình hành

).

Bài 11 : Cho nửa đường tròn (O) đường kính AB, M thuộc cung AB, C thuộc OA. Trên nửa mặt phẳng bờ AB
có chứa điểm M, kể các tiếp tuyến Ax, By với (O). Đường thẳng qua M vuông góc với MC cắt Ax, By lần lượt
tại P và Q, AM cắt CP tại E, B cắt CQ tại F.
a) Chứng minh tứ giác APMC, EMFC nội tiếp.
b) Chứng minh EF//AB.
c) Tìm vị trí của C để tứ giác AEFC là hình bình hành.
d) Cho EC.EP= FC.FQ. Chứng minh: EC=PQ và EP=FC

Hướng dẫn giải

y
x
a) Tứ giác APMC, EMFC nội tiếp. Q

Tứ giác APMC có :


Tứ giác APMC nội tiếp được đường tròn. M
Tương tự, Tứ giác EMFC nội tiếp được đường tròn.
b) EF//AB. P
F

Ta có : (Góc nội tiếp, góc tạo bới tia tiếp E


tuyến và dây cung cùng chắn cung MP).
(Góc nội tiếp, góc tạo bới tia tiếp tuyến và A C O B
dây cung cùng chắn cung AM).

Lí luận tương tự : (1)


Do đó : = 1800.

(Tổng các góc trong ).

Tứ giác MECF nội tiếp (Hai góc nội tiếp cùng chắn cung MF) (2)

Từ (1) và (2) ta có: , mà hai góc này ở vị trí đồng vị EF//AB.


c) Tìm vị trí của C để tứ giác AEFC là hình bình hành

Do EF//AB nên để tứ giác AEFC là hình bình hành

AE//CF cân tại C

MC=AC=CB .
d) EC=PQ và EP=FC

15
Nhóm file Word toán THCS

Vì EF//AB , áp dụng định lí Ta-let ta có:

Do : EC.EP=FC.FQ (gt)

Vì và có (đối đỉnh), (Hai góc nội tiếp cùng chắn cung AP)
(g.g).

Tương tự : .

Từ (3) và (4) ta có .

Xét và có:
(cùng phụ );
(cmt);
(góc trong bằng góc ngoài tại đỉnh đối diện của tứ giác nội tiếp).

(g.c.g).

mà (gt) .

Bài 12. Cho nửa đường tròn đường kính , thuộc cung , thuộc đoạn thẳng . Trên nửa mặt
phẳng bờ có chứa điểm kẻ các tia tiếp tuyến , với . Qua kẻ đường thẳng vuông góc
cắt tại . Qua dựng một đường thẳng vuông góc cắt tia tại . Gọi là giao điểm và ;
là giao điểm của và .

a) Chứng minh tứ giác và nội tiếp.

b) Chứng minh .

c) Chứng minh ba điểm , , thẳng hàng.

d) Chứng tỏ rằng hai đường tròn ngoại tiếp hai tam giác và tiếp xúc nhau tại .

e) Cho cố định, khi chuyển động trên nửa đường tròn thì trung điểm của chuyển động trên
đường nào?

Hướng dẫn giải

16
Nhóm file Word toán THCS
a) Tứ giác và nội tiếp.
(góc nội tiếp chắn nửa đường tròn)

Tứ giác nội tiếp.


Tương tự

Nên tứ giác nội tiếp.


b) .
Ta có
(hai góc nội tiếp cùng chắn ).
(hai góc cùng chắn )
(góc nội tiếp, góc tạo bởi tia tiếp tuyến và dây cùng chắn )
, mà hai góc ở vị trí đồng vị nên .
c) Ba điểm , , thẳng hàng.
Ta có
(hai góc tiếp cùng chắn )
(đồng vị, )
(góc nộ tiếp, góc tạo bởi tiếp tuyến và dây cùng chắn ).
, và là hai đỉnh liên tiếp của tứ giác nên nội tiếp (cùng thuộc một
cung chứa góc).

Do đó .
Nên ba điểm , , thẳng hàng.
d) Đường tròn ngoại tiếp hai tam giác và tiếp xúc nhau.
Dựng tiếp tuyến của đường tròn ngoại tiếp .
(góc nội tieps và góc tạo bởi tia tiếp tuyến và dây cùng chắn )
(cùng phụ với hai góc bằng nhau)
cũng là tiếp tuyến của đường tròn ngoại tiếp tại (định lý đảo về tiếp tuyến).
Vậy đường tròn ngoại tiếp hai tam giác và tiếp xúc ngoài tại .
e) Trung điểm của chuyển động trên đường tròn nào?
là đường trung bình của hình thang ( vì cùng vuông góc với ) tia
tại , nằm cùng nửa mặt phẳng bờ có chứa tia .

Bài 13. Cho đoạn thẳng . Trên cùng nửa mặt phẳng bờ là đường thẳng , ta vẽ nửa đường tròn
tâm đường kính và hai tiếp tuyến , với . Một đường thẳng thay đối cắt , lần lượt
tại các điểm , ( , ). Gọi là giao điểm của và .

a) Chứng minh rằng nếu là tiếp tuyến của thì .


17
Nhóm file Word toán THCS

b) Chứng minh rằng nếu thì đường thẳng là tiếp tuyến của .

c) Cho tiếp xúc với tại . Tìm vị trí của để tứ giác nội tiếp được trong đường tròn.

d) Trường hợp tiếp xúc với . Gọi là bán kính đường tròn nội tiếp tam giác . Chứng minh rằng:

Hướng dẫn giải


a) .
Nếu là tiếp tuyến của đường tròn thì , lần lượt là hai tia phân giác của hai góc kề bù
và .
b) Giả sử thì là tiếp tuyến của .
Giả sử , từ vẽ tiếp tuyến với đường tròn cắt tại . Dựa vào kết quả câu a) ta có
suy ra là tiếp tuyến của đường tròn .
c) Định vị trí của để tứ giác nội tiếp.

Ta có suy ra tứ giác là hình thang.

Mà nội tiếp được trong đường tròn là hinh thang cân cân tại .

là trung điểm (kẻ ). Đặt .


Áp dụng tính chất hai tiếp tuyến cắt nhau, ta có

vuông tại ta có

Vậy cần tìm là qua cách một khoảng bằng và tiếp xúc với đường tròn .

18
Nhóm file Word toán THCS

d)
Gọi là đường tròn nội tiếp lần lượt tiếp xúc với , , tại , , .
Tứ giác là hình vuông (vì , ).
Ta có

Trong có

Ta lại có

Từ (1), (2) và (3) ta được .

Bài 14. Cho đoạn thẳng có trung điểm là . Trên cùng nửa mặt phẳng bờ dựng nửa đường tròn
đường kính và nửa đường tròn đường kính . Trên lấy điểm (khác và ), tia
cắt tại , gọi là giao điểm thứ hai của với .

a) Chứng minh tam giác cân.

b) Tiếp tuyến tại của cắt tia tại , xác định vị trí tương đối của đường thẳng đối với và
.

c) Đường thẳng cắt tại , đường tròn ngoại tiếp cắt tại điểm thứ hai là . Chứng minh
ba điểm , , thẳng hàng.

d) Tại vị trí của sao cho , hãy tính độ dài đoạn thẳng theo .

Hướng dẫn giải.

19
Nhóm file Word toán THCS
a) Tam giác cân.
Ta có cân tại mà
(góc nội tiếp chắn nửa đường tròn ) .
cũng là đường phân giác , nghĩa là

(hai góc ở tâm bằng nhau nên cung chắn


bằng nhau)
cân tại .
b) Vị trí của đối với và .
và có: chung; (cmt);

(c.g.c) hay
tại , là bán kính là tiếp
tuyến của và .
c) , , thẳng hàng
Giả sử cắt đường tròn tại .
Ta có (góc ở tâm và góc nội tiếp cùng chắn )
(cmt)
, mà , là hai đỉnh liên tiếp của tứ giác .
Tứ giác nội tiếp (vì cùng thuộc một cung chứa góc)
Vậy là giao điểm thứ hai của với . Vậy , , thẳng hàng.

Bài 14. Cho đoạn thẳng có trung điểm là . Trên cùng nửa mặt phẳng bờ dựng nửa đường tròn
đường kính và nửa đường tròn đường kính . Trên lấy điểm (khác và ),
tia cắt tại , gọi là giao điểm thứ hai của với
a) Chứng minh tam giác cân.
b) Tiếp tuyến tại cắt tia tại xác định vị trí tương đối của đường thẳng đối với và .
c) Đường thẳng cắt tại , đường tròn ngoại tiếp cắt tại điểm thứ hai là . Chứng
minh ba điểm thẳng hàng.
d) Tại vị trí của sao cho , hãy tính độ dài đoạn thẳng theo .
Hướng dẫn giải

20
Nhóm file Word toán THCS

C N

E
D M

A O' K O B

a) cân
Ta có: cân tại .
Mà (góc nội tiếp chắn nửa đường tròn )
.
cũng là đường phân giác , nghĩa là
(hai góc ở tâm bằng nhau nên cung chắn bằng nhau)
cân tại .

b) Vị trí của đối với và


và có: chung; (cmt);
(c.g.c) hay tại , là bán kính
là tiếp tuyến của và

c) thẳng hàng
Giả sử cắt đường tròn tại
Ta có:
(góc ở tâm và góc nội tiếp cùng chắn )
(cmt)
, mà là hai đỉnh liên tiếp của tứ giác
Tứ giác nội tiếp (vì cùng thuộc một cung chứa góc)
Vậy là giao điểm thứ hai của với
thẳng hàng.

d) Tính theo
Dựng
Vì (gt) (so le trong)

21
Nhóm file Word toán THCS

mà (cmt)
cân tại
Mặt khác:
(vì cùng vuông góc )
(gt) và
Tứ giác là hình chữ nhật (hình bình hành có một góc vuông)
Đặt
Ta có:

(nhận)

(loại)

Vậy .

Bài 15. Cho đường tròn tâm có đường kính . Gọi là điểm di động trên đường tròn .
Điểm khác , dựng đường tròn tâm tiếp xúc với tại Từ và kẻ hai tiếp tuyến
và với đường tròn tâm vừa dựng.

a) Chứng minh lần lượt là các tia phân giác của các góc và .

b) Chứng minh ba điểm nằm trên tiếp tuyến của đường tròn tâm tại điểm .

c) Chứng minh không đổi, từ đó tính tích theo .d\

d) Giả sử ngoài trên nửa đường tròn đường kính không chứa có một điểm cố định. Gọi là
trung điểm của kẻ vuông góc với . Khi chuyển động thì chuyển động trên đường cố định
nào.
Hướng dẫn giải

22
Nhóm file Word toán THCS

C
M

P
D

I
1 2
2 1
A B
O H
K

a) là phân giác và
Do là hai tiếp tuyến cắt nhau đối với đường tròn
là tia phân giác

Lý luận tương tự: là tia phân giác

b) nằm trên tiếp tuyến tại của


(góc nội tiếp chắn nửa đường tròn)

Vậy , mà (gt) nên thẳng hàng.


Ta có: là đường trung bình của hình thang vuông nên mà
(gt) tại , là bán kính của là tiếp tuyến của đường tròn
tại

c) không đổi và tính theo


Áp dụng tính chất hai tiếp tuyến cắt nhau của một đường tròn, có

23
Nhóm file Word toán THCS

(const)
Áp dụng hệ thức lượng trong tam giác vuông:

(do vuông có là trung tuyến ứng với cạnh huyền)

d) Điểm chuyển động trên đường nào


Ta có: (vì cùng vuông góc với ). Kéo dài cắt tại .
có là đường trung bình là trung điểm mà cố định nên cố định.
Điểm luôn nhìn hai điểm cố định dưới một góc vuông nên chuyển động trên đường
tròn đường kính

Bài 16. Cho nửa đường tròn đường kính , điểm thuộc nửa đường tròn. Gọi là điểm chính giữa
cung , là giao điểm của và . Gọi là giao điểm của và .
a) Chứng minh rằng
b) Gọi là điểm đối xứng với qua . Chứng minh là tiếp tuyến của .
c) Chứng minh rằng:

d) Nếu . Gọi là giao điểm của và . Chứng minh:

Hướng dẫn giải

a)
Ta có: (góc nội tiếp chắn nửa đường
tròn)

Tương tự:
có hai đường cao cắt nhau tại
.
là trực tâm
(tính chất ba đường cao)
b) là tiếp tuyến của
Do là điểm chính giữa
(hai góc nội tiếp cùng chắn hai cung bằng nhau.)
Mà (hai góc nội tiếp cùng chắn )

24
Nhóm file Word toán THCS

có là đường cao đồng thời là đường trung tuyến ( và đối xứng qua )
cân tại

Ta có:
tại
là tiếp tuyến của
c)

Ta có:

Do đó:
d)


có vừa là đường cao vừa là đường phân giác
cân tại nên cũng là đường trung trực

Ta có:


Suy ra

Bài 17. Cho đường tròn đường kính , điểm thuộc đường tròn . Trên nửa
mặt phẳng bờ có chứa điểm , kẻ tia tiếp xúc với đường tròn . Gọi là điểm chính giữa cung
nhỏ . Tia cắt tại , tia cắt tại .
a) Chứng minh các tam giác và cân.
b) Khi , tính theo .
c) Khi là điểm chính giữa nửa đường tròn , kẻ đường và là điểm bất kỳ thuộc cung nhỏ .
Xác định vị trí của để tích đạt giá trị lớn nhất.

25
Nhóm file Word toán THCS
Hướng dẫn giải

a) và cân x
Do là điểm chính giữa
Q
(hai góc nội tiếp chắn hai cung bằng
nhau)
N
là đường phân giác trong
C
Mặt khác (góc nội tiếp chắn nửa đường tròn)
M
có vừa là đường cao vừa là đường phân giác
cân tại
Ta lại có: (vì cùng bù ). Do đó B A
O
cân tại .
b) Khi , tính theo

Do (gt) cân tại

(vì cùng bù với hai góc bằng nhau)


(g-g)

(do cân tại nên )


 0
; BCA  90 (góc nội tiếp chắn nửa đường tròn)
Xét
BAQ vuông tại A , AC  BQ có:
AB 2  BC.BQ  BC  BN  NQ   BC  AB  BC  (1)
Đặt BC  x, x  0 , biết AB  2 R , từ (1) cho
4 R 2  x  2 R  x   x 2  2 Rx  4 R 2  0
 '  R2  4R2  5R2   '  R 5
x1   R  R 5 và x2   R  R 5 < 0 (loại)

Vậy
BC   5 1 R 

26
Nhóm file Word toán THCS
c) Tính M để tích P  MA.MB.MC.MD đạt giá trị lớn nhất C
Gọi H , K lần lượt là hình chiếu của M trên OA, OC .
K M
Ta có:
P  MA.MB.MC.MD
 2 S MAB .2 S MCD (vì  
AMB  CMD  900 )
 AB.MH .CD.MK  4 R 2 .MH .MK B
O A
H
 4 R 2 .MH .OH (vì MK  OH )
2 2 2
2 MH  OH 2 OM
 4R .  4R .  2R4
2 2
4
Vậy P  2 R D
Đẳng thức xảy ra  MH  OH  M là điểm chính giữa

AC .
Do đó maxP = 2R4.

Bài 18: Cho đường tròn (O;R), đường kính AB. M là điểm thuộc đường tròn (O). Vẽ đường tròn tâm O1 tiếp
xúc với đường tròn (O) ở M và tiếp xúc với AB ở N. AM và BM cắt đường tròn (O1) tại điểm thứ hai là C và D.
a) Chứng minh ba điểm C,O1, D thẳng hàng.
b) Chứng minh đường thẳng MN luôn đi qua một điểm cố định K khi đường tròn (O1) thay đổi.
c) Gọi giao điểm của các tia CN và DN với BK và AK lần lượt là C’ và D’. Chứng minh: C’D’=NK.
d) Xác định vị trí của điểm O1 để chu vi tam giác NC’D’ đạt giá trị nhỏ nhất.
Hướng dẫn giải
a) C,O1, D thẳng hàng.

AMB  900 ( góc nội tiếp chắn nửa đường tròn)

M
 CMD  900
Suy ra CD là đường kính của đường tròn tâm O1 C D
Suy ra ba điểm C,O1, D thẳng hàng. O1
b) MN luôn đi qua một điểm cố định.
Đường tròn (O) và (O1) tiếp xúc tại M suy ra M,O1, O A B
N O
thẳng hàng.
MO1 D và MOB cân tại O và O D'
1
 
 O1MD  O1 DM  MBO
mà hai góc ở vị trí đồng vị
  C'
nên CD//AB  NC  ND

 NCM 
 NMD  450  AMK  KMB  450 K
Vậy K là điểm chính giữa của cung AB của đường tròn (O) nên K cố định.
c) C’D’=NK.
  1  0
 NAD '  sd KB  45
 2  AD ' N
  
AND '  DNB  NMD  45 0
Ta có:  vuông cân tại D’.
Lý luận tương tự:  NC ' B vuông cân tại C’
27
Nhóm file Word toán THCS
   0
Vậy D ' NC  ND ' K  NC ' K  90  tứ giác D’NC’K là hình chữ nhật.

Suy ra C’D’=NK.
d) Xác định vị trí của điểm O1 để chu vi tam giác NC’D’ đạt giá trị nhỏ nhất.
Gọi P là chu vi tam giác NC’D’, ta có:

P = ND’+C’D’+NC’=BC’+C’K+NK=KB+NK=R 2 +NK

 R 2  OK  R 2  R  2  1 R 
Dấu “=” xảy ra  N  O  O1 là trung điểm của OK’ (trong đó KK’ là đường kính của (O)).
Vậy khi O1 là trung điểm của OK’ (KK’ là đường kính của (O)) thì chu vi tam giác NC’D’ đạt giá trị
nhỏ nhất.
Bài 19: Cho đường tròn (O;R) đường kính AC. Trên đoạn OC lấy điểm B, gọi M là trung điểm của AB. Dựng
dây cung DE vuông góc với AB tại M. Từ B kẻ BF vuông góc với CD (F thuộc CD).
a) Chứng minh tứ giác BMDF nội tiếp được đường tròn.
b) Chứng minh: CB.CM=CF.CD.
c) Chứng minh ba điểm B,E,F thẳng hàng.
DA DB DE
 
d) Gọi S là giao điểm của BD và MF, CS lần lượt cắt DA,DE tại R và K. Chứng minh: DR DS DK
Hướng dẫn giải
a) Chứng minh tứ giác BMDF nội tiếp được đường D
tròn.
Do DE vuông góc với AC tại M, BF vuông góc với
CD (gt)
  0 R F
nên BMD  DFB  180 suy ra tứ giác K
S
BMDF nội tiếp được đường tròn đường kính BD. I H
b) CB.CM=CF.CD. A
M
C
O B
CF CM
cos 
ACD    CB.CM  CF .CD J
BC CD
c) Ba điểm B,E,F thẳng hàng.
Trong đường tròn (O) có AC vuông góc
với DE tại M auy ra MD=ME (đường kính và dây
cung).
E
Tứ giác ADBE có: MD=ME, MA=MB (gt)
và AB  DE suy ra ADBE là hình thoi
Suy ra EB//AD.

 0
Ta có: ADC  90 (góc nội tiếp chắn nửa đường tròn)
 AD  DC
 BF / / AD (vì cùng vuông góc với DC).

Treo tiên đề Ơclit, từ B ngoài AD ta chỉ kẻ được một đường thẳng song song với BD nên B.E,F thẳng
hàng.

28
Nhóm file Word toán THCS
DA DB DE
 
DR DS DK
d)
Vẽ AJ, BI cùng song song với CR (I,J thuộc DE).
AMJ  BMI ( AM  MB; AMJ    900 , M
 BMI  AJ  MBI
 )
 MI  MJ
Áp dụng định lý TaLet ta có:
DA DB DJ  DI  DM  MJ    DM  MI  2DM DE
    
DR DS DK KD DK DK
Cách khác:
Gọi H là giao điểm của EF và CK.
EH EK EB  BH EK EB BH EK
     
Do EH//DR: DR DK DR DK DR DR DK
DA EK BH
 
Vì EB=AD nên DR DK DR (1)
SB BH SB  DS BH DB BH
   1   1
Do BH//DR: DS DR DS DR DS DR (2)
DA DB EK BH  BH  EK DE
      1  1 
Từ (1) và (2) DR DR DK DR  DR  DK DK

Bài 20: Cho đường tròn (O;R) đường kính AC. Trên đoạn thẳng OC lấy điểm B và vẽ đường tròn (O’) có
đường kính BC. Gọi M là trung điểm của AB, qua M kẻ dây cung vuông góc với AB cắt đường tròn (O) tại D
và E. Nối CD cắt đường tròn (O’) tại I.
a) Tứ giác DAEB là hình có đặc tính gì? Vì sao?
b) Chứng minh: MD=MI và MI là tiếp tuyến của đường tròn (O’).
c) Gọi H là hình chiễu của I trên BC. Chứng minh : CH.MB = BM.MC.
Hướng dẫn giải
a) Tứ giác DAEB là hình gì ? D
Đường kính AC vuông góc với dây DE tại M => M = ME.
Tứ giác ACDE có MD = ME, MA = MB (gt), AB 
DE, suy ra  : Tứ giác ACDE
I
là hình thoi ( Hình bình hình có hai đường chéo vuông góc
với nhau).
b) MD = MI và MI là tiếp tuyến của (O’) A B C
0
Ta có :  BIC  90 ( Góc nội tiếp chắn nữa đường tròn M O H O'

(O’))
ADC  900 ( Góc nội tiếp chắn nữa đường tròn
(O))
Suy ra : BI  CD, suy ra : E, B, I thẳng hang theo tiên
đề Ơclít. E
Tam giác DIE có IM là trung tuyến ứng với cạnh
huyền, suy ra : MI = MD.
Do MI = MD (cmt)  Tam giác MDI cân tại M, suy
ra : MID  MDI .
O’I = O’C = R’  Tam giác O’IC cân tại O’  O ' IC  O ' CI .
29
Nhóm file Word toán THCS
0
Suy ra: MID+O ' IC  MDI  O ' IC  90 (Tam giác MCD vuông tại M)
Vậy MI  O’I tại I. O’I = R’ bán kính của đường trong (O’)
Suy ra : MI là tiếp tuyến của (O’).

c) Chứng minh : CH.MB = BM.MC.


Ta có : BCI  BIM *(Góc nội tiếp, góc tạo bơit tiếp tuyến và dây cùng chắn cung BI).
Ta lại có: BI C  BIH => IB là phân giác của góc MIB của tam giác MIB.
Mặt khác: BI  CI=> IC là phân giác ngoài tại đỉnh I của tam giác MIH
Áp dụng tính chất dfdường phân giác đối với tam giác MIH có :
BH IH CH
   CH .MB  BH .MC
MB MI CM

Bài 21: Cho tam giác ABC đều, dựng nữa đường tròn tâm D đường kinhd BC tiếp xức với AB,AC lần lượt tại
K và L. Lấy P thuộc cung nhỏ PL, Dựng tiếp tuyến với nưa đường tròn tại P cắt các cạnh AB, AC lần lượt tại M
và N.

a) Chứng minh rằng : , rồi suy ra : .

b) Chứng minh rằng: .


c) Gọi E, F lần lượt là các điểm trên các cạnh AB, Ac sao cho chu vị bằng một nửa chu vi tam giác
. Chứng minh rằng: .
Hướng dẫn giải
A A

N N

P P
E E

K' L' K L

B C C
D D

BC 2
BM .CN 
a) BMD ∽ CDN rồi suy ra: 4
0 0 0 0 0
Xét tứ giác : AKDL có : KDL  KAL  180 ( Do K  L  90 )  KDL  180  60  120
Theo tính chất hai tiếp tuyến cắt nhau ta có: DM, DN lần lượt là tia phân giác của KDP và
KDP  PLD KLD 1200
PLD  MDN   
2 2 2
30
Nhóm file Word toán THCS
0
MDC  MDN+NDC  60  NDC
 0
Ta có : MDC  B  BMD  60  BMD
0
Suy ra : NDC  BMD , mà BMD=DCN  60 ( Tam giác ABC đều)
BM BD BC 2
 BMD CDN    BM .CN  BD.CD 
CD CN 4
S MDN MN

b) S ABC 2 BC .
1
S MDN 2 MN .PD MN PD MN KD
  .  .
S ABC 1 BC AD BC 2 BC
AD.BC
Ta có: 2

Vì D  MD là tia phân giác BMN  DK  DP
  900 , KAD
 AD KD 1
K  300  KD   
AKD có 2 AD 2
 0
c) EDF  60 .

Dựng đường tròn bàng tiếp trong góc A có tâm O của AEF . Do AD là đường trung tuyến của ABC đều

nên AD là tia phân giác BAC . Suy ra: O  AC
Gọi P’, K’, L’ lần lượt là tiếp điểm của (O) với EF, AB, AC. Ta có:
AK '  AL ', P ' E  EK ', P ' F  FL ' (tính chất hai tiếp tuyến cắt nhau)
1
PAEF  PABC ( gt )
Mà 2
1 3
 2 AK '  PABC  AB (ABC
2 4 đều)
3 AB
 AK '  AB  BK ' 
4 4 (vì AK ' K ' B  AB )
2
AB
 BK '. AB 
4
2
 BC  BC 2
BD 2    
Mặt khác:  2  4 (D là trung điểm BC)
AB  BC (ABC đều)
 BK '. AB  BD 2  BK ' D BDA(c.g .c)

 BKD   900
 BDA

Ta lại có OK ' B  90  O  D (vì O, D  AD )
0

  0 
Mà K ' AL '  K ' DL '  180 ( AK ' DL ' là tứ giác nội tiếp) mà K ' AL '  60
0

' DL '  1200  EDF


K   600 (tia phân giác của hai góc kề)

Bài 22. Cho đường tròn đường kính AB. Trên đường tròn (O) lấy điểm P sao cho . Tiếp tuyến
tại P với đường tròn (O) cắt tia AB tại T.
31
Nhóm file Word toán THCS

a) Chứng minh rằng: và .


b) Hạ tại H. Chứng minh rằng: .
c) Cho Tính .
Hướng dẫn giải
2   0
a) TP  TB.TA và BTP  2TPB  90 P
TBP và TPA có:    TAB
ATP chung, TPB 
(góc tạo
 1
bởi tia tiếp tuyến và dây, góc nội tiếp cùng chắn PB ) 2
TB TP A
 TBP ∽ TPA    TP 2  TB.TA
TP TA . O
T B H
  1   1
TPB BP 
Ta có: 2 sđ 2 sđ BOP
  2TPB
 BTP   BTP  BOP  900 (OPT vuông tại
P)

HB. AT  BT .HA
b)
  PAB
P  (cmt ); P
  PAB
  
Ta có 1 2 (cùng phụ HPA )  PB là tia phân giác TPH trong TPH .
 0
Mà PA  PB( BPA  90  góc nội tiếp chắn nửa đường tròn)
 PA là phân giác ngoài tại đỉnh P của TPH
Áp dụng tính chất phân giác trong và phân giác ngoài đối với TPH có
BH PH HA
   BH . AT  BT .HA
TB TP AT
S PTB
c) Tính
400
PH 2  HB.HA  400  3HA  HA 
3
AB 1 1  400  409
R   HA  HB     3 
2 2 2 3  6
391 1227
HB. AT  BT .HA(cmt )  3  AB  BT   BT .  409  BT 
3 391
1 1 1227 12270
S BPT  BT .PH  . .20  (
Vậy 2 2 391 391 đvdt).

Bài 23. Cho tam giác ABC có nội tiếp đường tròn . Các tiếp tuyến của đường tròn (O) tại A,
C cắt nhau tại M. BM cắt đường tròn (O) tại D. Chứng minh rằng:

a) .
b) .
c)
d) Tam giác CBD cân.

32
Nhóm file Word toán THCS
Hướng dẫn giải
MA AD

MB AB
a)
 
Xét MAD và MBA có chung, MAD  MBA (Góc nội tiếp, góc tạo bởi tia tiếp tuyến và dây cung

cùng chắn AD )
MA AD MD
 MAD ∽ MBA( g.g )   
MB AB MA
AD.BC  AB.CD
b)
MD MD
 
Ta có MA  MC (tính chất hai tiếp tuyến cắt nhau của một đường tròn) MA MC
MD CD B

Lập luận tương tự, ta có: MC BC . Suy ra:
AD CD A
  AD.BC  AB.CD
AB BC .
AB.CD  AD.BC  AC.BC E O
D
c)
 
Dựng điểm E  AC sao cho: EDC  ADB
DAB và DEC có:  ADB  EDC
(cách dựng), M
 
ADB  ECD (hai góc nội tiếp cùng chắn 
AD ) C
 DAB DEC ( g.g )
AB DB
   AB.DC  EC.BD (1)
EC DC
   
Do EDC  ADB  BDC  ADE , nên DAE ∽ DBC ( g .g )
 AD.BC  BD. AE (2)
Từ (1), (2) cho: AB.CD  AD.BC  BD( AE  EC )  BD. AC
CBD
d) cân
 AD.BC  AB.CD
  2 AB.CD  AC.BD
Ta có:  AD.BC  AB.CD  AC.BD
Mà AC  2 AB( gt )  2 AB.CD  AC.BD  CD  BD
Suy ra BCD cân tại D.
Bài 24. Trên nửa đường tròn tâm, đường kính AB, lấy hai điểm M, E theo thứ tự A, M, E, B. Hai đường thẳng
AM và BE cắt nhau tại C, AE và BM cắt nhau tại D.
a) Chứng minh rằng: Tứ giác MCED nội tiếp và CD vuông góc với AB.

b) Gọi H là giao điểm của CD và AB. Chứng minh rằng: .


c) Chứng minh rằng các tiếp tuyến tại M và E của đường tròn (O) cắt nhau tại một điểm I thuộc CD.

d) Cho Tính diện tích tam giác ABC theo R.

33
Nhóm file Word toán THCS
Hướng dẫn giải
a) Tứ giác MCED nội tiếp và CD vuông góc với AB. C
Áp dụng tính chất góc nội tiếp chắn nửa đường tròn, ta có:

AEB  
AMB  900 , vậy
 M I
BMC 
AEC  900
E
 AEC  BMC  1800
 tứ giác MCED nội tiếp đường tròn D
ABC có hai đường cao BM, AE cắt nhau tại D  D là trực
tâm ABC  CD  AB.
b) BE.BC  BH .BA . A O H B
BE BH
cos 
ABC    BE.BC  BH . AB
AB BC
c) Các tiếp tuyến tại M và E của đường tròn (O) cắt nhau tại một điểm I thuộc CD.
Gọi I là giao điểm của tiếp tuyến tại M của đường tròn (O) với CD.
  
Trong đường tròn (O) có: IMD  MAB (góc nội tiếp, góc tạo bởi tia tiếp tuyến và dây cung chắn MB )
  
MAB  MDI (cùng phụ với ACH )
 
 IMD ADI  IMD cân tại I  IM  ID
 
Ta lại có: IMC  ICM (cùng phụ với hai góc bằng nhau)
 MIC cân tại I  IM  IC . Vậy IM  ID  IC  I là trung điểm của CD.
CED có EI là trung tuyến ứng với cạnh huyền nên IE  IC  ID  IM
MIO và IEO có: IM  IE (cmt ), OI chung, OM  OE  R.
 IMO  IEO(c.c.c)  IEO   IMO  900  IE  OE , OE  R
nên IE là tiếp tuyến của đường tròn
(O) tại E. Nghĩa là các tiếp tuyến tại M, E của đường tròn (O) cắt nhau tại điểm I thuộc CD.
S ABC
d) Tính theo R.
  900 , CAH
  450  AHC vuông cân tại H  CH  AH  x
AHC có H
  300  EBA
EAB   600

  HB  cot g 600  3  HB  3 .x
cot g EBA
HC 3
3 6R
AB  AH  HB  2 R  x  xx  R (3  3)
Ta có 3 3  3
AB.CH 1
S ABC   .2 R.R (3  3) R 2
Vậy 2 2 (đvdt)
Bài 25. Cho tam giác ABC nhọn, dựng nửa đường tròn tâm O đường kính BC. Qua A vẽ các tiếp tuyến AP, AQ
với đường tròn (O), P và Q là tiếp điểm (Q, B nằm cùng phìa với OA). Gọi H là trực tâm tam giác ABC, M là
giao điểm của PQ và AO, K là giao điểm của AH và BC. Chứng minh rằng:
a) Tứ giác MHKO nội tiếp được đường tròn.
b) Ba điểm P, H, Q thẳng hàng.

34
Nhóm file Word toán THCS
c) Các đường thẳng AK, BQ, CP đồng quy.
Hướng dẫn giải
a) Tứ giác MHKO nội tiếp đường tròn T

Ta có: AP = AQ ( tính chất hai tiếp tuyến cắt nhau)


OP = OQ = R
 OA là trung trực của PQ  AO  PQ (1) A'
A
 0 2
APO có P  90 , PQ  AO; AP  AM . AO
S
ASH và AKC có A chung, P
M
H
 H
AS AKC  900  ASH ∽ AKC ( g.g ) Q
N
AS AH
   AS. AC  AK . AH B K O
C
AK AC

Tương tự: APS ∽ ACP(g.g)


AP AS
   AS.AC=AP 2 2
AC AP Do đó AH . AK  AP  AM . AO
AH AO
 

AM AK Mà HAM chung  AHM ∽ AOK (c.g.c)
  0
Suy ra: AMH  AKO  90 => Tứ giác MHKO nội tiếp đường tròn
 0
b) Dựa vào kết quả câu a, ta có: HMO  90  AM  HM (2)
Từ (1) và 92) , ta có : P, H, Q thẳng hàng.
c) Gọi N là giao điểm của CQ và BP, T là giao điểm của CP và AQ; A’ là trung điểm của TN

BPC 
 BQC  900 ( Góc nội tiếp chắn nửa đường tròn)

 BTC có hai đường cao CQ và BP cắt nhau tại N


 N là trực tâm BTC  TN  BC
PTN vuông tại P có PA’ là trung tuyến ứng với cạnh huyền  A ' T  A ' P  A ' PT cân tại A’
A ' PT  
A ' TP (3)
 
Ta có: OP = OC= R  OPC cân tại O  OPC  OCP (4)
  0   0
Do BC  TN (cmt )  PTA '  OCP  90  A ' PT  OPC  90 (theo (3) và (4))


A ' PO  900  A ' P  OP tại P, OP = R nên A’P là tiếp tuyến của đường tròn (O).
Tương tự: A’x cũng là tiếp tuyến của (O) tại Q

35
Nhóm file Word toán THCS
Suy ra: A  A ' mà AH  BC  H  TN  CP, BQ, AK đồng quy.
Bài 26: Cho đường tròn (O;R) có hai đường kính AB và CD vuông góc, Gọi S là một điểm bất kỳ trên đoạn
OB. Tia CS cắt (O) tại M.
a) Chứng tỏ O, S, M, D cùng thuộc một đường tròn. Xác định tâm I của đường tròn.
b) Tiếp tuyến tại M của (O) cắt AB , CD lần lượt tại E và F. Chứng tỏ ES = EM
và CS.CM = 2ME.MF
c) Đường tròn (I) cắt EF tại Q . Trong trường hợp S là trung điểm của OB, tính SOQF theo R.

Hướng dẫn giải



CMD  900 A
a) Ta có: ( Góc nội tiếp

 0
chắn nửa đường tròn), SOD  90 ( gt )
F
Nên O, M thuộc đường tròn đường O
D
C

kính SD có tâm I là trung điểm SD.


I

 O, S , M , D cùng thuộc đường tròn (I) S


Q


EMS 
 CDM B
M

E
b) ( Góc tạo bởi tia tiếp tuyến và

dây cung và góc nôi tiếp cùng chắn một cung CM)

ESM 
 CDM ( Góc trong bằng góc ngoài tại đỉnh đối diện của tứ giác nội tiếp)

 EMS  M  ESM
 ES cân tại S  ES=EM

 OC CM
cos DCM    CS.CM  OC.CD  2 R 2  2.OM 2
Ta có: CS CD (1)
 0 2
Mặt khác: OEF có OEF  90 , OM  EF  OM  ME.MF (2)

Từ (1) và (2) ta có: CS .CM  2 ME.MF


OMQ
c) vuông tại M nội tiếp đường tròn (I) => OQ là đường kính của (I)

 I là trung điểm của OQ

Mặt khác OC = OD = R => OI là đường trung bình của CSD  OI / / CS


FO OQ
OQ / / CM  FOQ ∽ FCM  
Nghĩa là: FC CM

36
Nhóm file Word toán THCS
2R2 2R2 4R R 2 5R 2
CM    (CS 2  OC 2 +OS2  R 2   )
CS 5 5 4 4
Từ (1): 2

5
OQ  SD  CS  R
2

FO 5 4R 5 FO 5 5
 R:     FO  R
Vậy: FC 2 5 8 FC  FO 3 3

1 1 5 R 5R 2
SOQF  OF.DQ= . R. 
Suy ra: 2 2 3 2 12 (đvdt).

Bài 27. Cho đường tròn (O) nội tiếp tam giác ABC tiếp xúc với các cạnh BC, CA, AB theo thứ tự tại D, E, F .
Đường vuông góc với OC ở O cắt hai cạnh CA, CB lần lượt tại I, J. Một điểm P di động trên cung nhỏ DE
không chứa điểm F, tiếp tuyến tại P của đường tròn (O) cắt hai cạnh CA, CB lần lượt tại M, N.

a) Cho , hãy xác định theo các góc của tam giác ABC.

b) Chứng minh ba tam giác IMO, OMN và JON đồng dạng. Suy ra .
c) Đảo lại, nếu M, N là hai điểm trên CE, CD thỏa mãn (*) thì MN tiếp xúc với đường tròn tâm O.

Hướng dẫn giải


C

P 1 2 D
M 2
1
E J

O
I

A F B

a) Áp dụng tính chất hai tiếp tuyến của đường tròn cắt nhau, ta có:

 M
M   N
N 
1 2 và 1 2

 1  1
MOP  EOP PON  POD
Do đó: 2 và 2

37
Nhóm file Word toán THCS
   1   )  1 DOE

 MON  MOP  PON  ( EOP  POD
2 2

1 
 ) 0 ACB
 (1800  DCE   0  90 
2 ( Do tứ giác CEOD có E  D  180 ) 2 (không đổi).

CIJ CIJ
b) có OC vừa là đường cao vừa là đường phân giác nên cân tại C.


CIO   900  ACB  MIO
  CJO   OJ
 N  MON

Nên: 2

   
IMO và OMN có: M1  M 2 (cmt ); MIO  MON  φ  IMO ∽ OMN ( g.g )

   
OMN và JON có: N1  N 2 (cmt ); NJO  MON  φ  JON ∽ OMN ( g .g )

Vậy: IMO ∽ OMN ∽ JON


IM IO
  IM.JN  IO.OJ  OI 2  OJ 2 (*)
Suy ra: JO JN
IM JO
IM .JN  JO.JO  
M  CE ; N  CD IO JN
c) Đảo lại, nếu thỏa mãn hệ thức (*) thì: OI = OJ và mà
  OJ
MIO  N (cmt )  IMO ∽ JON (c.g .c)  OMI
 N  OJ

      
Từ đó ta có: MON  NOJ  MOJ  OMI  MIO  NOJ  MIO

ACB

 MON   900 
 MIO
2
Ta chứng minh được MN là tiếp tuyến của (O). Thật vậy, qua M vẽ tiếp tuyến thứ hai ( khác ME) với
đường tròn (O) cắt Cd tại điểm N1 , theo chứng minh phần thuận ta có:

ACB

 MON 0  
1  90   MON 1  MON
2 , tia ON, ON1 nằm cùng phía với OM nên hai tia ON, ON1
trùng nhau nên N trùng N1 => MN tiếp xúc (O).

Bài 28. Cho đường tròn (O;R) và điểm A ở ngoài đường tròn với OA = 2R, Từ A dựng hai tiếp tuyến AB’,AC’
với (O). Dựng tia đối của tia OA cắt đường tròn (O) tại A’. Dựng tiếp tuyến với đường tròn (O) tại A’ lần lượt
cắt các tia AC’, AB’ tại B và C.
a) Chứng minh tam giác ABC đều và tính diện tích tam giác ABC theo R.
b) Gọi M là điểm bất kỳ trên cung nhỏ B’C’ và H, K, L lần lượt là hình chiếu của M trên cạnh BC, AC, AB.
Gọi J là giao điểm của MH và B’C’. Chứng minh rằng các tứ giác JMKB’, LMJC’ nội tiếp được đường tròn.

38
Nhóm file Word toán THCS

c) Chứng minh rằng: .

d) Chứng minh rằng: .

Hướng dẫn giải


 
a) Ta có: C ' AO  B ' AO (tính chất hai tiếp tuyến A
cắt nhau).

' AO  O 'C R 1
sin C    sin 300
OA 2 R 2 K

L M
' AO  B
C ' AO  300  C
' AB '  600
C' B'
J I

B H A' C

Email: tuthanhlinh123@gmail.com
Facebook: https://www.facebook.com/thienthannho007

Vì BC là tiếp tuyến của


O  tại A nên AA  BC .

Vậy ABC có AA là đường cao và là đường phân giác đồng thời C AB  60 nên ABC đều.
0

3R 3
AC  AA tan 
AAC  AA tan 300   R 3  BC  2 R 3
3 .
BC 2 3
S ABC   3R 2 3
Vậy 4 .
b) Tứ giác JMKB, LMJC  nội tiếp
 
Ta có MJB  MKB  180 nên tứ giác JMKB nội tiếp.
0

Tương tự tứ giác LMJC  nội tiếp.


2
c) MJ  MK .ML
  
Ta có MJL  MC L (hai góc nội tiếp cùng chắn ML ).
 
MC L  MBC  (Hệ quả của góc tạo bởi tiếp tuyến và dây).
 C   MKJ
MB  
(Hai góc nội tiếp cùng chắn MJ ).
  MKJ
 MJL 
.

39
Nhóm file Word toán THCS
 
Lý luận tương tự ta có  MLJ  MJK
MJ ML
   MJ 2  MK .ML
Suy ra MJL ∽ MKJ (g.g) MK MJ .
d) MH  MK  ML

Giao điểm của AA với BC  là I , ta có: S AMB  S AMC  S BMC   S ABC
MK . AB ML. AC  MJ .BC  AI .BC 
   
2 2 2 2
Mà AB  AC   BC  (tam giác ABC  đều)
3R
AI  AI 
Suy ra ML  MK  MJ  AI (do 2 )
Ta có
MH  MK  ML  MH  MK  ML  2 MK .ML
 MJ  JH  MK  ML  2 MK .ML
 MJ  MK  ML  MJ  MK  ML  2 MK .ML
 2 MJ  2 MK .ML
 MJ 2  MK .ML
Do đó MH  MK  ML .

Bài 29. Cho đường tròn và điểm ở ngoài đường tròn sao cho . Từ dựng hai tiếp tuyến

với đường tròn .


a) Chứng minh tứ giác là hình vuông.

b) Tia cắt đường tròn tại ( thuộc cung nhỏ ). Tính theo .

c) Lấy thuộc cung lớp , dựng tiếp tuyến với tại cắt các tia lần lượt tại các điểm sao
cho . Dựng đường thẳng vuông góc với tại trung điểm của cắt tia phân giác góc
tại . Chứng minh tứ giác nội tiếp.
d) Đường thẳng đi qua P, Q cắt đường cao AH của tam giác ABC tại M , tia FI cắt AB tại E . Chứng minh
rằng AE  AM .
Lời giải
a) Tứ giác APIS là hình vuông

40
Nhóm file Word toán THCS

2 2 2 2
Áp dụng Pitago: SA  IA  IS  R  SA  R
Mặt khác SA  AP (tính chất hai tiếp tuyến cắt nhau)  SA  AP  IP  IS  R

ASI  900 (do AS tiếp tuyến)  Tứ giác APIS là hình vuông.
b) Tính SK
Gọi O là giao điểm hai đường chéo hình vuông
R 2
OI  OS 
2 và 
AOS  900
R 2
2 2 2
OS 
KS  OK  OS với 2
R 2 R
OK  IK  OI  R 
2

2
2 2  
R2 R2
   
2
 KS 2  2 2   R2 2  2
4 2  SK  2  2 R
c) Tứ giác QICN nội tiếp
 0
Ta có NF  AC tại F , FA  FC nên tam giác ANC cân tại N có NAC  45 suy ra tam giác ANC vuông
cân tại N .
  0
Suy ra INC  90 mà IQC  90 ( BC là tiếp tuyến tại Q )
0

  0
Suy ra tứ giác QICN nội tiếp đường tròn  IQN  ICN  180 (1).
d) AE  AM
Gọi R là giao điểm của NF và BC
  0
Ta có BPI  BQI  180 nên tứ giác BPIQ nội tiếp đường tròn.

  ABC
  IBP
IQP 
2 (hai góc nội tiếp cùng chắn cung IP
Suy ra )
  
Mà BRN  RNC  RCN (góc ngoài tam giác RNC )

ABC  BRN
(so le trong)

RNC 
 FCN (tam giác FNC cân tại F )

41
Nhóm file Word toán THCS
       
  FCN  RCN  ICN  FCI  RCN  ICN  RCI  RCN  ICN
IQP 
Suy ra 2 2 2 (2).
  0
Từ (1) và (2) cho IQN  IQP  180 suy ra P, Q, N thẳng hàng.
Ta có NF //SI //AE (cùng vuông góc với AC )
IQ //AM (cùng vuông góc BC )
SI FS IN IQ
  
Áp dụng định lí talet AE FA NA AM mà SI  IQ  R  AE  AM .

Bài 30. Cho đường trong đường kính , gọi là điểm đối xứng của qua . Từ vẽ hai tiếp

tuyến với đường tròn .


a) Chứng minh tứ giác nội tiếp. Xác định tâm và bán kính đường tròn ngoại tiếp.

b) Tiếp tuyến tại với lần lượt cắt tại . Chứng minh
c) Gọi là tâm đường tròn ngoại tiếp tam giác và là điểm bất kì trên cung . Trên lấy điểm
sao cho . Chứng minh tam giác đều và tổng không ớn hơn đường kính của đường tròn
tâm .
Lời giải

a) Tứ giác SMON nội tiếp. Xác định tâm và bán kính
 
Ta có SMO, SNO cùng nhìn OS dưới một góc vuông nên M , N thuộc đường tròn đường kính OS tâm A bán
kính OA  R .
2
b) SA  AC.SM
 OM R 1 
sin OSM     OSM  30 0
OS 2 R 2
  0
Vậy CSD  2OSM  60
Tam giác SAC có

42
Nhóm file Word toán THCS
A  900 ;  SC
ASC  300  AC 
2
SA  SC  AC  SC  CM 2
2 2 2 2

  SC  CM  SC  CM   SM  SC  AC   SM . AC
c) IS  IC không lớn hơn đường kính của đường tròn tâm O
  0 
Tam giác IHC có IC  IH , CIH  CSD  60 (Hai góc nội tiếp cùng chắn cung CD )
Suy ra tam giác IHC đều nên CH  IC
 
Xét tam giác CSI và tam giác CDH có CH  IC , CS  CD, ICS  HCD
 CSI  CDH  c.g .c   IS  HD
.

Ta có IS  IC  HD  IH  ID  DD ( DD là đường kính của


 O  )
Bài 31. Cho tam giác đều, gọi là trung điểm của . Các điểm lần lượt di động trên các cạnh
sao cho .
a) Chứng minh tích không đổi.
b) Chứng minh tia là tia phân giác của .
c) Dựng đường tròn tâm tiếp xúc với . Chứng minh đường tròn này luôn tiếp xúc với .

d) Gọi lần lượt là tiếp điểm của với . lần lượt là giao điểm của với .
Chứng minh .
Lời giải

a) BD.CE không đổi



 BDO 
 BOD   1200
 1800  B


BOD 
 COE 
 180  DOE  1200
Ta có 
BD OB
    O
 
Suy ra BDO  COE mà DOE  B  60  BDO đồng dạng COE OC CE

43
Nhóm file Word toán THCS

BC 2
=> BD.CE = BO.CO = 4 ( không đổi)

b) DO phân giác BDE
OD BD BD
    0
∆BDO đồng dạng với ∆COE => OE OC BO mặt khác DBO=DOE=60
 
=> ∆BDO đồng dạng với ∆ODE (c.g.c) => BDO=ODE , mà tia DO nằm giữa hai tia DB và DE

=> DO là tia phân giác của BDE
c) Đường tròn (O) luôn tiếp xúc với DE, AC

∆ABC đều nên đường trung tuyến AO cũng là đường phân giác trong của BAC , mà DO là phân giác ngoài tại
đỉnh D=> O là tâm đường tròn bàng tiếp trong góc A của ∆ADE => Đường tròn (O) luôn tiếp xúc với DE, AC.
d) DE = 2IN
AP = AQ ( tính chất hai tiếp tuyến cắt nhau), AB =AC
AP AQ
   0 
=> AB AC => PQ //BC => IQA=ACB  60 mà DOE=60
0

  0
=> IQE=IOE=60 , O, Q là hai đỉnh liên tiếp của tứ giác IOQE
=> Tứ giác IOQE nội tiếp ( cùng thuộc một cung chứa góc)
  0
Suy ra: EIO  EQO  90
 0
Lý luận tương tự: DNE  90
 
Vậy tứ giác DINE ( DIE và DNE cùng nhìn DE dưới 1 góc vuông)
 
=> ONI  ODE . Vậy ∆ONI đồng dạng với ∆ODE(g.g)
IN ON 1
  cos600 
=> DE OD 2 => DE = 2 NI.

Bài 32. Cho tam giác ABC cân tại A. Các cạnh AB, BC, CA tiếp xúc đường tròn (O) lần lượt tại các điểm
D, E, F.
a) Chứng minh rằng : DF song song với BC và A, O, E thẳng hàng .
b) Gọi giao điểm thứ hai của BF với đường trong (O) là M và N là giao điểm của DM với BC. Chứng
minh rằng: ∆DNB đồng dạng với ∆BFC và NB = NE
c) Gọi (O’) là đường tròn đi qua ba điểm B, O, C. Chứng minh rằng: AB, AC là các tiếp tuyến của
đường tròn (O’)
44
Nhóm file Word toán THCS

Hướng dẫn giải


a) DF//BC và A, O, E thẳng hàng A
Áp dụng tính chất hai tiếp tuyến cắt nhau tại một điểm, ta có:
AD AF

AD = À và AO ⊥ DF. Vậy AB AC (∆ABC cân tại A nên AB
= AC) => DF // BC ( định lý Ta – lét đảo)
BC tiếp xúc với (O) tạo E nên OE⊥BC. Từ O ngoài BC ta chỉ kẻ F
D
được một đường thẳng vuông góc với BC nên A, O, E thẳng
hàng.
M
E C
B N

O'

b) ∆BFC đồng dạng với ∆DNB và N là trung điểm của BE


Ta có:

 MBN  (so le trong, DF//BC)
 BFN


 DFM 
 BDM  )
( goùc noäi tieáp, goùc taïo bôûi tia tieáp tuyeán vaø daây suøng chaén MD

 
=> MBN  BDM
   
∆BFC và ∆DNB có: ABC  ACB (gt), MBN  BDM (cmt)
=> ∆BFC đồng dạng với ∆DNB (g.g)
BN BD BE 1
  
=> FC BC BC 2 ( do ∆ABC cân tại A nên đường phân giác AE cũng là trung tuyến)
1 1 1
BN  FC  EC  BE
=> 2 2 2 ( do CE, CF là tiếp tuyến của (O) nên CE =CF)
Vậy N là trung điểm của BE.
c) AB, AC là các tiếp tuyến của đường tròn (O’)

Áp dụng tính chất hai tiếp tuyến BD, DE cắt nhau tại một điểm, ta có BO là phân giác của DOE =>

DOB  (1)
 BOE

 
Mà O’B = OO’( bán kính đường tròn (O’)) => ∆O’OB cân tại O’ => EOB  OBO '(2)
 
Từ (1) và (2) ta có: DOB  OBO ' , mà hai góc ở vị trí so le trong => OD//O’B

45
Nhóm file Word toán THCS
Ta lại có: AB ⊥OD ( AB là tiếp tuyến (O)) => AB ⊥O’B tại B, O’B là bán kính của đường tròn (O’) =>
AB là tiếp tuyến của đường tròn (O’).
Lý luận tương tự : OE//O’C, mà AC⊥OE=> AC ⊥O’C tại C, O’C là bán kính của đường tròn (O’). Vậy
AC là tiếp tuyến của đường tròn (O’).

Bài 33. Cho đường tròn (O;R), điểm A cố định trên đường tròn, kẻ tiếp tuyến Ax với (O). Điểm M tùy ý trên
Ax, vẽ tiếp tuyến thứ 2 MB với đường tròn (O). Gọi I là trung điểm của MA, K là giao điểm của BI với đường
tròn (O), MK cắt đường tròn (O) tại C.
a) Chứng minh rằng: ∆MIK đồng dạng với ∆BIM và BC //MA.
b) Tìm vị trí của M để AMBC là hình bình hành.
c) Chứng minh khi M di động trên Ax thì trực tâm H của tam giác MAB thuộc 1 đường tròn cố định.

Hướng dẫn giải


a) ∆MIK đồng dạng với ∆BIM và BC //MA M
  IBA

∆IAK và ∆IBA có: AIB chung, IAK ( góc nội tiếp, góc tạo bởi tia
tiếp tuyến và dây cùng chắn 
AK )

=> ∆IAK đồng dạng với ∆IBA (g.g)


I
IK IA K
  IA2  IK .IB
=> IA IB B
H
Vì I là trung điểm của MA nên E
A
IM IB O

IA = IM => IM2 = IK.IB=> IK IM
C

IM IB


=> ∆MIK và ∆BIM có: MIK chung, IK IM (cmt)
Suy ra ∆MIK đồng dạng với ∆BIM (c.g.c)
    
=> IMK  IBM , ta lại có IBM  BCK ( góc nội tiếp, góc tạo bởi tia tiếp tuyến và dây cùng chắn BK )
 
=> IMK  BCK , mà hai góc này ở vị trí so le trong nên BC //MA.

46
Nhóm file Word toán THCS

Bài 33. Cho đường tròn


O; R  , điểm A cố định trên đường tròn, kẻ tiếp tuyến Ax với O  . Điểm M tùy ý
trên Ax , vẽ tiếp tuyến thứ hai MB với đường tròn
O  . Gọi I là trung điểm MA. K là giao điểm của BI với
đường tròn
O  , MK cắt đường tròn O  tại C.
a) Chứng minh rằng MIK ∽ BIM và BC song song MA .
b) Tìm vị trí điểm M để AMBC là hình bình hành.
c) Chứng minh khi M di động trên Ax thì trực tâm H của tam giác MAB thuộc một đường tròn cố định.

Hướng dẫn giải


a) MIK ∽ BIM và BC // MA .
IAK và IBK có:
AIB chung,
  IBK (góc nội tiếp, góc tạo bởi tiếp tuyến và dây cung M
IAK

cùng chắn AK )
 IAK ∽ IBA (g.g)
IK IA
   IA2  IK .IB
IA IB .
I
Vì I là trung điểm của MA nên IA  IM .
K
2 IM IB
 IM  IK .IB   B
IK IM H
Khi đó, MIK và BIM có: E

IM IB A

 -chung, IK IM (cmt)
MIK O

 MIK ∽ BIM (c.g.c) C



 IMK 
 IBM  
, ta lại có IBM  BCK (góc nội tiếp, góc tạo

BK
bởi tiếp tuyến và dây cung cùng chắn ).
  BCK

 IMK , mà hai góc ở vị trí so le trong nên BC / / MA .

b) Xác định M  Ax để AMBC là hình bình hành.


 
Tứ giác AMBC là hình bình hành  AMB  ACB
1 .
  
Mà ACB  MBA (góc nội tiếp và góc tạo bởi tia tiếp tuyến và dây cung chắn AB )
 
(1)  AMB  MBA  MAB cân tại A.
 MAB đều (do MA, MB là hai tiếp tuyến cắt nhau của đường tròn O  nên MA  MB )
 AMBC 
ACB  60o
Tứ giác là hình thoi có .
47
Nhóm file Word toán THCS
 ABC đều

AOB  2 
AOE  2 
ACB
Do
R 3

AOE  60o  AE  OA sin 60o 
2 .
 AB  AM  2 AE  R 3 .

Vậy M nằm trên tia Ax sao cho AM  R 3

c) Trực tâm H của MAB thuộc đường tròn cố định:


Do H là trực tâm MAB nên AH  MB và BH  MA

{
 AH /¿ O B(cùng vuông góc với MB)
AH /¿ OA (cùng vuông góc với MA )
Mà OA  OB  R .
 Tứ giác HABO là hình thoi  AH  OA  R .
 H   A; R 

A cố định, R thay đổi nên đường tròn  A; R  cố định. Vậy trực tâm của H của MAB thuộc một đường tròn
cố định.

Bài 34. Cho đường tròn và điểm A ở bên ngoài đường tròn. Vẽ hai tiếp tuyến với đường tròn

( là các tiếp điểm). Gọi là trung điểm .

a) Chứng minh tứ giác nội tiếp và xác định tâm I của đường tròn này.
b) Chứng minh rằng: .
c) Gọi là trọng tâm của tam giác . Chứng minh: .
d) Chứng minh: vuông góc .

Hướng dãn giải

48
Nhóm file Word toán THCS

M
E

G' I A
O G
H

a) Tứ giác ABOC nội tiếp và xác định tâm I của đường tròn
AB, AC là hai tiếp tuyến cắt nhau của đường tròn O  nên 
ABO  
ACO  B, C thucộc đường tròn
Do
đường kính OA có tâm I là trung điểm của OA .

b) AM . AO  AB. AI
AB
AM . AO  .2 AI  AB. AI
Ta có 2
c) MG // BC
GE 1

Gọi E là trung điểm của MA G
, do là trọng tâm CMA nên G  CE và CE 3.
ME 1 MA MB BE
 ME   ME 
Mặt khác : BE 3 (vì 2 2 nên 3 )
GE ME

Suy ra: CE BE , theo định lí Ta let đảo  MG / / BC
d) IG  CM
Gọi G ' là giao điểm của OA và CM  G ' là trọng tâm ABC .
G ' M 1 GE
 
Do đó, CM 3 BE , theo định lí Ta –let đảo GG '/ / ME 1 .
MI là đường trung bình trong OAB  MI / / OB , mà AB  OB (cmt)
 MI  AB , nghĩa là MI  ME . (2)

Từ
1 và  2  cho: MI  GG ' , ta lại có GI '  MK (vì OA  MK ) nên I là trực tâm MGG '
 GI  G ' M tức GI  CM .
49
Nhóm file Word toán THCS

Bài 35. Qua điểm A ở ngoài đường tròn (O; R), kẻ cát tuyến ABC với đường tròn (O) (B nằm giữa A và C). Các
tiếp tuyến của đường tròn (O) tại B, C cắt nhau tại K. Qua K kẻ đường thẳng vuông góc với OA, cắt OA tại H
và cắt đường tròn (O) lần lượt tại E và F (E nằm giữa K và F). Gọi M là giao điểm của OK và BC. Chứng minh
rằng:
a) Tứ giác KBOC nội tiếp. Xác định tâm của đường tròn ngoại tiếp tứ giác này.
b) KB2 = KE.KF.
c) Tứ giác EMOF nội tiếp.
d) AE, AF là tiếp tuyến của đường tròn (O).
Hướng dẫn giải
a) KBOC nội tiếp. Xác định tâm của đường tròn
ngoại tiếp tứ giác này
Ta có BK, CK là tiếp tuyến (O) nên

OBK 
 OCK  900
 Tứ giác OBKC nội tiếp đường tròn đường kính OK,
có tâm là trung điểm OK.
b) KB2 = KE.KF
Xét ∆KBE và ∆KFB có:

BKF  
chung, KBE  BFK
(góc tạo bởi tia tiếp tuyến và dây,

góc nội tiếp cùng chắn BE )
 ∆KBE ∽ ∆KFB ( g.g)
KB KE

 KF KB  KB2 = KE.KF
c) EMOF nội tiếp
Dựa vào tính chất hai tiếp tuyến của đường tròn, ta có OK  BC
 0
∆KOB có B  90 , BM  OK nên BK2 = KM.KO
KM KF

Suy ra: KE.KF = KM.KO  KE KO
Vậy ∆KME ∽ ∆KFO (c.g.c)
 
 EMK  OFK , nên tứ giác EMOF nội tiếp (góc trong bằng góc ngoài tại đỉnh đối diện)
d) AE, AF là tiếp tuyến của (O)
 
Do OE = OF = R  ∆OEF cân tại O  OFE  OEF .
  
AME  900  EMK

 
AOE  900  OEF 
 900  OFE
Ta có:  .
   
Mà OFE  EMK (cmt) AME  AOE , nên tứ giác AOME nội tiếp (E, M là đỉnh liên tiếp của tứ giác nên
cùng thuộc một cung chứa góc)
 0  0
Ta lại có: AMO  90  AEO  90 . Vậy AE  OE, OE = R  AE là tiếp tuyến (O; R). Do
AFO   AEO  900
 AF cũng là tiếp tuyến (O; R).

50
Nhóm file Word toán THCS
Bài 36. Cho đường tròn (O; R), một dậy cung CD có trung điểm H. Trên tia đối của tia DC lấy điểm S. Qua S
kẻ tiếp tuyến SA, SB với đường tròn (O), đường thẳng AB cắt các đường thẳng SO, OH lần lượt tại E, F.
a) Chứng minh rằng: OE.OS = R2 và tứ giác SEHF nội tiếp đường tròn.
b) Biết R = 10cm, OH = 6cm, SD = 4cm. Tính độ dài các đoạn thẳng CD, SA.
c) Chứng minh rằng: Khi S di động trên tia đối của tia DC thì đường thẳng AB đi qua một điểm cố định và
có số đo không đổi.
Hướng dẫn giải
a) OE.OF = R2 và tứ giác SEHF nội tiếp
Áp dụng tính chất hai tiếp tuyến cắt nhau,
có SA = SB mà OA = OB = R
 SO là trung trực của AB
 OS  AB.
 0
∆OAS có A  90 , AE  OS
 OE.OS = OA2 = R2 .
  0
Ta có: FHS  FES  90
 Tứ giác SEHF nội tiếp đường tròn đường kính SF

b) Tính CD, SA
Áp dụng định lý Pytago đối với ∆OHD vuông tại H
HD  OD 2  OH 2  102  62  8(cm)
OH  CD tại H  HD = HC (định lý đường kính và dây cung)
 CD = 2HD = 16 (cm)
SH = SD + HD = 4 + 8 = 12 (cm)
OS2 = OH2 + SH2 = 62 + 122 = 180 (định lý Pytago đối với ∆SOH vuông)
SA  OS 2  OA 2
(định lý Pytago đối với ∆AOS vuông)
= 180  100  4 5(cm)

c) AB đi qua điểm cố định và CED có số đo không đổi.
  0 
∆HOS và ∆EOF có OHS  OEF  90 ,SOF chung
 ∆HOS ∽ ∆EOF (g.g)
OH OS

 OE OF  OH.OF = OE.OS = R2.
R2
OF 
Vậy OH .
 0   0
Ngoài ra, ta lại có E nằm trên OS, (E nằm giữa O, S vì SAE  90  SAO ) và HOS  90 nên F  tia OH, suy
ra F cố định.
Vậy AB đi qua điểm cố định F.
OH OD
 
Ta có: OH.OF = R = OD  OD OF , mà FOD chung.
2 2

 ∆FOD ∽ ∆DOH (c.g.c).


51
Nhóm file Word toán THCS
  0
 FDO  DHO  90 và D thuộc đường tròn đường kính OF.
  1 sdCFD
CED 
Trong đường tròn này sđ 2 (không đổi).
Bài 37. Cho đường tròn (O; R), qua điểm K ở bên ngoài đường tròn, kẻ các tiếp tuyến KB, KD (B, D là các tiếp
điểm), kẻ cát tuyến KAC (A nằm giữa K và C).
a) Chứng minh rằng: ∆KDA ∆KCD.
b) Chứng minh rằng: AB.CD = AD.BC.
c) Gọi I là trung điểm BD. Chứng minh rằng: Tứ giác AIOC nội tiếp.
d) Kẻ dây CN // BD. Chứng minh ba điểm A, I, N thẳng hàng.
Hướng dẫn giải
a) ∆KDA ∽ ∆KCD

Xét ∆KDA và ∆KCD có: CKD chung

KDA 
 KCD (góc tạo bởi tia tiếp tuyến
và dây, góc nội tiếp cùng chắn 
AD )
 ∆KDA ∽ ∆KCD (g.g)
KA AD

Vậy KD DC
b) AB.CD = AD.BC
Tương tự ∆KBA ∽ ∆KCB (g.g)
KA AB

 KB BC
KA KA AD AB
  
Do KD = KB (tính chất tiếp tuyến) nên KD KB DC BC
Do đó AB.CD = AD.BC
c) Tứ giác AIOC nội tiếp
Do OK lầ đường trung trực của BD, mà IB = ID nên I  OK
Mặt khác KB2 = KA.KC (∆KBA ∽ ∆KCB)
KB2 = KI.OK (hệ thức lượng trong tam giác vuông KOB)
KA KO

 KA.KC = KI.KO  KI KC
 ∆KAI ∽ ∆KOC (c.g.c)
 
 KAI  KCO , nên tứ giác AIOC nội tiếp (góc trong bằng góc ngoài tại đỉnh đối diện)
d) A, I, N thẳng hàng
Ta có: OA = OC = R  ∆OAC cân tại O
  
 OAC  OCA  KIA
  
Mà: OIC  OAC (hai góc nội tiếp cùng chắn OC )
 
 KIA  OIC
Tứ giác BCND nội tiếp (O) có CN // BD  BCND là hình thang cân.
Ta lại có OK là đường trung trực của BD nên OK cũng là đường trung trực của CN

52
Nhóm file Word toán THCS

Suy ra: IC = IN  ∆ICN cân tại I, do đó OI là tia phân giác CIN
  
 OIC  OIN  KIA
    0
Ta có: AIO  OIN  AIO  KIA  180  A, I, N thẳng hàng.

Bài 38. Cho ∆ABC có ba góc nhọn nội tiếp đường tròn (O; R), các tiếp tuyến tại B và C của (O) cắt nhau tại N.
Vẽ dây cung AM // BC; đường thẳng MN cắt đường tròn (O) tại điểm thứ hai là P. Đoạn thẳng AP cắt BC tại D.
a) Chứng minh tứ giác ABCM là hình thang cân.
b) Chứng minh rằng DB = DC rồi suy ra ba điểm O, D, N thẳng hàng.

c) Giải sử . Hãy tính độ dài BC.


Hướng dẫn giải
a) ABCM là hình thang cân
Do AM // BC (gt)  ABCM là hình thang
mà ABCM nội tiếp đường tròn (O)
 ABCM là hình thang cân
b) DB = DC

∆BPN và ∆MBN có BNM chung,
  NMB
NBP 
(hệ quả của góc tạo bởi tia tiếp tuyến và
dây cung)
 ∆BPN ∽ ∆MBN (g.g)
BP BN

 MB MN (1)

Tương tự: ∆NCP ∆NMC (g.g)
CP NC

 MC MN (2)
mà NB = NC(tính chất hai tiếp tuyến cắt nhau), kết
hợp với (1) và (2) ta có:
BP CP

MB MC (3)

BP CP

Do ABCM là hình thang cân nên MB  AC, MC  AB và từ (3) cho AC AB (4)
BDP và ADC có: BDP  ADC   
(đối đỉnh); PBD  DAC (hai góc nội tiếp cùng chắn PC )
BP BD
 BDP ∽ ADC (g.g)  
AC AD .
CP DC BD DC
ADB ∽ CDP (g.g)   .   DB  DC
Tương tự: AB AD Thay vào (4) ta được AD AD
Mặt khác: NB  NC (cmt); OB  OC  R  O, D, N thuộc đường trung trực của BC  O, D, N thẳng
hàng.
c) Tính BC

53
Nhóm file Word toán THCS
1 1 1 1 1
2
 2
 2
 2

Ta có: OB NC OB BN 64
1 1 1 1 1
     BD 2  64  BD  8.
Do BC  ON nên OB BN Vậy BC  2BD  16 .
2 2 2 2
BD BD 64

Bài 39. Cho tam giác ABC vuông tại A, có đường cao . Hạ vuông góc AB, vuông
góc AC. Đặt

a) Gọi r và lần lượt là bán kính đường tròn nội tiếp tam giác ABC và tam giác AHK. Hãy tính tỷ số theo x,
y, suy ra giá trị lớn nhất của tỷ số đó.
b) Chứng minh rằng tứ giác BKHC nội tiếp trong một đường tròn. Tính bán kính của đường tròn đó theo x, y.
Hướng dẫn giải
'
r
a) Tính r theo x, y và suy ra giá trị lớn nhất của
tỷ số
AHA1K
Tứ giác là hình chữ nhật (vì
  
HAK  AHA1  A1KA  900 )
AA1
Gọi I là giao điểm của và HK
Ta có: IA  IK
 IAK cân tại I
  IKA
 IAK 
 
Mà ABC  IAK (hai góc cùng phụ với góc IAB)
 
Do đó: AKH  ABC

Xét ABC và AKH có: BAC (chung)

ABC 
 AKH
Do đó: ABC ∽ AKH

r ' HK
'

r, r lần lượt là bán kính đường tròn nội tiếp tam giác ABC, AKH nên chứng minh được r BC
AA1
Mặt khác: ABC vuông tại A; là đường cao nên:
2
AA1  A1B.A1C  x.yAA1  xy. Vậy HK  xy
r' xy

Ta có: r x  y
xy xy xy 1
xy   
Áp dụng bất đẳng thức Cô-si cho hai số dương x, y ta có: 2 nên x  y 2(x  y) 2
r' 1

Vậy r 2 . Dấu “=” xảy ra  x  y  ABC vuông tại A

54
Nhóm file Word toán THCS
r' 1

Vậy giá trị lớn nhất của tỷ số r 2
b) Chứng minh tứ giác BKHC nội tiếp. Tính bán kính đó theo x, y.

AKH 
 HKC  1800 (hai góc kề bù)

ABC 
 AKH (chứng minh trên)
  0
Do đó ABC  HKC  180
 Tứ giác BHKC nội tiếp đường tròn
KA1
Vẽ cắt đường tròn (BHKC) tại D (D  K)

DCK  900  DC là đường kính của đường tròn (BHKC)

 CBD  900
 
Ta có: HB / / KD  HK  BD  HK  BD
BD  HK  xy
Do đó:
BDC vuông tại B, theo định lý Pitago, ta có:

 
2
DC 2  BC2  BD 2  (x  y) 2  xy  DC  x 2  3xy  y 2
1 2
x  3xy  y 2
Vậy bán kính của đường tròn ngoại tiếp tứ giác BHKC là: 2 .
Bài 40. Cho nhọn nội tiếp đường tròn (O; R). Tiếp tuyến của đường tròn (O) tại B cắt các tiếp tuyến của
đường tròn (O) tại A và C lần lượt ở M và N. Kẻ đường cao BH của .

a) Chứng minh HB là đường phân giác của góc .



b) Kẻ phân giác của góc ABC cắt đường tròn (O) tại D. Dựng đường tròn (D, DA) cắt các đường thẳng BA,
BC lần lượt tại các điểm Q, P. Chứng minh BO vuông góc PQ.
c) Gỉa sử ON  2R , gọi E, F lần lượt là trung điểm ON, BN. Đường thẳng CF cắt đường tròn (O) lần nữa tại K.
Chứng minh KB vuông góc với KE.
Hướng dẫn giải

55
Nhóm file Word toán THCS


a) HB là phân giác MHN
Gọi I là giao điểm OM và AB, J là giao điểm của ON và BC
BC
 BJ  JC 
2 (tính chất hai tiếp tuyến cắt nhau)
  0  
JBN và HAB có H  J  90 , BAH  NBJ (hệ quả của góc tạo bởi tia tiếp tuyến và dây)
 JBN ∽ HAB (g.g)
AH AB AB.JB AB.BC
   AH  
JB BN BN 2BN (1)
AB.BC
HC 
Lý luận tương tự ta có: 2BM (2)
AH BM AM
 
Từ (1) và (2) cho: CH BN CN (theo tính chất tiếp tuyến có AM  BM; BN  CN )
  
Mà MAH  NCH (hai góc tạo bởi tia tiếp tuyến và dây cùng chắn ABC )
 AHM ∽ CHN (c.g.c)  AHM 
 CHN

 MHB 
 NHB (cùng phụ với hai góc bằng nhau)

56
Nhóm file Word toán THCS

Vậy HB là đường phân giác của MHN .
b) BO  PQ
  
Do BD là phân giác ABC  ABD  CBD
  DC
 AD   AD  DC
(hai cung bằng nhau nên hai dây bằng nhau)

 C  (D; DA). Do đó: AQP  ACP  
(hai góc nội tiếp cùng chắn AP )
 
Mặt khác: ACP  ABM (hệ quả của góc tạo bởi tia tiếp tuyến và dây)
  ABM
 AQP 
, mà hai góc ở vị trí so le trong nên PQ // MB.
Ta lại có: MB  OB (do MB là tiếp tuyến của (O) tại B)
 OB  PQ .
c) KB  KE


Theo tính chất hai tiếp tuyến của đường tròn cắt nhau, ta có: ON là phân giác BOC

BOC 1 

 BON   sñBC
2 2 (3)
Do E, F là trung điểm ON, BN nên EF là đường trung bình của OBN
 EF / / OB và EF  NB (vì NB  OB )
   
Vậy: BON  NEF (đồng vị), NEF  BEF (BE là trung tuyến ứng với cạnh huyền ON nên
BE  EN  BEN cân tại E)
  BON
 BEF 
(4)
  
Mặt khác BKF  BCK  CBK (góc ngoài tại đỉnh của BCK )
 1 
BCK  sñBK   
2 (góc nội tiếp chắn BKF  BON  BEF )

 1 
CBK  sñCK 
2 (góc nội tiếp chắn CK )

57
Nhóm file Word toán THCS
 1   1 sñBC
  sñCK) 
 BKF  (sñBK
2 2 (5)
  
Từ (3) và (4), (5) cho BKF  BON  BEF , mà K, E là hai đỉnh liên tiếp của tứ giác BKEF  tứ giác
BKEF nội tiếp (vì cùng thuộc một cung chứa góc)
  0
Suy ra: BKE  BFE  90  BK  KE

TRANG 67 71

Bài 41: Cho đường tròn (O;R) nội tiếp ABC , tiếp xúc với cạnh AB, AC lần lượt ở D và E.

a) Gọi O’ là tâm đường tròn nội tiếp ADE , tính OO’theo R.


b) Các đường phân giác trong của góc B và góc C cắt đường thẳng DE lần lượt tại M và N . Chứng minh tứ giác
BCMN nội tiếp được đường tròn.
MN DM EN
 
c) Chứng minh : BC AC AB .

Hướng dẫn giải


a) Tính OO’ theo R A
Gọi O’ là giao điểm của AO với cung nhỏ DE của
M
đường tròn (O)  O’ thuộc đường phân giác của góc A O' E
trong ADE
D
  N
Ta có: DOA  EOA ( tính chất hai tiếp tuyến cắt nhau)
 DO 'O 'E
O
 1  ' 1
ADO '   EDO  'E
Mà 2 sđ DO ' ; 2 sđ O
B
  '
ADO '  EDO .
C

 DO’ là phân giác của góc D  O’ là tâm đường tròn nội tiếp ADE . Do đó: OO’=R

b) Tứ giác BCMN nội tiếp đường tròn

Do AB= AC ( tính chất hai tiếp tuyến cắt nhau)  ADE cân tại A nên:

1800  BAC 
BAC

ADE   900 
2 2

ABC  
ABC

ADE   
ABM  NMB   NMB 
ABM 

Mà: 2 ( do BO là phân giác ABC nên 2 )


B  
BAC ABC ACB

 NMB 
ADE   900  
2 2 2

ACB

NCB 
Mặt khác: 2 ( do CO là tia phân giác 
ACB )
58
Nhóm file Word toán THCS
 
Suy ra NMB  NCB , mà M, C là hai đỉnh liên tiếp của tứ giác BCMN
 Tứ giác BCMN nội tiếp( vì cùng thuộc một cung chứa góc)

MN DM EN
 
BC AC AB
c)


NMO & BCO có: NOM 
 BOC  
(đ đ); NMO  BCO (cmt)
OM ON MN
 NMO ∽ BCO( g .g )   
OC OB BC
DM OM NE ON
DMO ∽ ACO( g.g )   & NEO ∽ BAO ( g .g )  
Tương tự: AC OC AB OB

MN DM EN
 
Vậy, BC AC AB

Bài 42: Cho đường tròn (O;R) và một điểm A ở ngoài đường tròn (O). Kẻ các tiếp tuyến AB, AC với đường
tròn (O) (B , C là các tiếp điểm). Gọi H là trung điểm BC
a) Chứng minh A, H, O thẳng hàng và các điểm A,B,O,C thuộc một đường tròn.
b) Kẻ đường kính BD của (O) , vẽ CK vuông góc với BD. Chứng minh rằng: AC.CD=CK.AO.
c) Tia AO cắt đường tròn (O) theo thứ tự tại M,N. Chứng minh: MH.AN=AM.HN.
d) AD cắt CK tại I. Chứng minh rằng I là trung điểm CK.

Hướng dẫn giải


a) A, H, O thẳng hàng và các điểm B
A,B,O,C thuộc một đường tròn.
Ta có: AB=AC( tính chất hai tiếp tuyến 1 2
cắt nhau); OB = OC =R; HB=HC(gt) 
A,H,O thuộc đường trung trực của BC, A M O N
nên A,H,O thẳng hàng. H
K
I

E C D

ABO & 
ACO cùng nhìn OA dưới một góc vuông nên A, B, O, C thuộc đường tròn đường kính OA.

b) AC.CD=CK.AO


BAC  900 (góc nội tiếp chắn nửa đường tròn)  CD  BC


 OA / / CD ( cùng vuông góc với BC)  CDK 
AOB ( đồng vị)

59
Nhóm file Word toán THCS
 CK AB AC
sin CDK  sin 
AOB  
Ta có CKD vuông tại K nên CD ; AO AO

CK AC
   AC.CD  AO.CK
CD AO
c) MH.AN=AM.HN

 0
Ta có: MBN  90 (góc nội tiếp chắn nửa đường tròn)

 BNM 
B 
2 ( vì cùng phụ với NBH )


BNM 
B 
Mà 1 ( góc nội tiếp và góc tạo bởi tiếp tuyến và dây cung chắn MB )
 B
B 
1 2 . Vậy BM là đường phân giác trong tại đỉnh B của AHB

Ta lại có : BN  BM  BN là đường phân giác ngoài tại đỉnh B của AHB

Áp dụng tính chất đường phân giác trong và đường phân giác ngoài của AHB có:
MH BH NH
   MH . AN  AM .NH
AM AB AN
d) I là trung điểm của CK

Gọi E là giao điểm của hai tia BA và DC

ABC cân tại A  


ABC  
ACB
Ta có: AB=AC nên
 
Suy ra: AEC  ACE (cùng phụ với hai góc bằng nhau)
 AEC cân tại A  AE  AC . Vậy AB=EA.

CK / / EB
(cùng vuông góc với BD), áp dụng hệ quả của định lý Ta lét ta có
IK ID CI
   IK  CI
AB DA EA

Suy ra I là trung điểm của CK.

Bài 43. Cho đường tròn (O;R) và một điểm A ở ngoài đường tròn sao cho OA=3R. Từ A vẽ hai tiếp tuyến AB,

AC đến đường tròn (O) với B, C là hai tiếp điểm.

a) Chứng minh tứ giác OBAC nội tiếp.

60
Nhóm file Word toán THCS
b) Từ B vẽ đường thẳng song song với AC cắt đường tròn (O) tại điểm D (khác điểm B. Đường thẳng AD cắt
đường tròn (O) tại E (khác điểm D) và tia BE cắt AC tại F. Chứng minh rằng F là trung điểm của AC.
c) Chứng minh tia đối của tia CE là tia phân giác góc BEA.
d) Gọi H là giao điểm của BC và OA. Chứng minh rằng HB là phân giác của góc EHD.

e) Tính diện tích BDC theo R.

Hướng dẫn giải

B x
a) Chứng minh tứ giác OBAC nội tiếp.
Do AB, AC là hai tiếp tuyến của (O) nên D
  OCA
OBA   90 E
  OCA
OBA   180
Ta có A
O H
Suy ra tứ giác OBAC nội tiếp.
b) Chứng minh rằng F là trung điểm của
F
AC.

C
FC FE
FCE ∽ FBC ( g .g )    FC 2  FB.FE (1)
FB FC

Ta có
  BDE
EAF  AC / / BD )
(So le trong,

BDE 
 FBA 
(góc nội tiếp và góc tạo bởi tia tiếp tuyến và dây cung cùng chắn BE )
FA FE
  FAE ∽ FBA( g .g )    FA2  FB.FE (2)
 EAF  FBA . Vậy FB FA
2 2
Từ (1) và (2) suy ra FA  FC  FA  FC  F là trung điểm của AC.
c) Tia Ex là tia phân giác góc BEA.

    
Ta có BCE  BDE (góc nội tiếp cùng chắn BE )  BCE  EAC

EBC 
 ECA 
(góc nội tiếp và góc tạo bởi tia tiếp tuyến và dây cung cùng chắn CE )
  CEA
 BCE ∽ CAE ( g .g )  BEC   BEx
  EAx

(cùng bù với hai góc bằng nhau). Suy ra Ex là tia
phân giác góc BEA.
d) Chứng minh HB là phân giác của góc EHD
AB AE
ABE ∽ ADB( g .g )    AB 2  AD.AE
AD AB
61
Nhóm file Word toán THCS
 2
ABO có B  90, BH  OA, AB  AH . AO
AE AO
 AD.AE  AH.AO   
AH AD và ODA chung
 
Suy ra AEH ∽ AOD(c.g.c)  AHE  ADO (3)
Suy ra tứ giác OHED nội tiếp (có góc trong bằng góc ngoài tại đỉnh đối diện)
 
Ta lại có OD=OE=R  ODE cân tại O  ODE  OED (4)

OED 
 OHD 
(góc nội tiếp cùng chắn OD ) (5)
   
Từ (3), (4), (5) suy ra AHE  OHD  DHB  EHB (cùng phụ hai góc bằng nhau)
Vậy HB là phân giác của góc EHD.

e) S BDC theo R .

Do AC / / BD ( gt), OC  AC  OC  BD  CBD cân tại C

AB  AC  ABC cân tại A , mà  


ACB  DBC ( so le trong và BD / / AC )
2
S  BC 
 CBD   
 ACB ∽ CBD (g.g) S ACB  AC 

Áp dụng hệ thức lượng trong tam giác vuông có:

OB 2 R 2 R 8R
OB 2  OH .OA  OH    HA  OA  OH 
OA 3R 3 và 3

R 8 R 8R 2 2 2R
BH 2  OH .HA  .   BH 
3 3 9 3

4 2R
BC  2 BH 
3

AC 2  OA2  OC 2  9 R 2  R 2  8R 2

2 2 R 8R 16 2 R 2
S ACB  2S BHA  BH .HA  . 
3 3 9 ( đvdt)

62
Nhóm file Word toán THCS
2
 BC  32 R 2 1 16 2 R 2 64 2 R 2
SCBD  .S
 ACB  . . 
Vậy  AC  9 8R 2 9 81 ( đvdt).

Bài 44. Cho đường tròn


O; R  đường kính DD ' , gọi B là điểm nằm ngoài đường tròn O  sao cho D ' là
trung điểm OB . Từ B vẽ hai tiếp tuyến BA, BC đến đường tròn O  ( với A, C là các tiếp điểm)

a) Chứng minh tứ giác ABCD là hình thoi. Tính diện tích ABCD theo R .

b) Gọi M là điểm bất kỳ trên cạnh AB . Các đường thẳng DM và BC cắt nhau tại N . Chứng minh hệ thức :
AB 2  AM .CN

c) Gọi E là giao điểm của tia CM với AN . Chứng tỏ tứ giác AEBC nội tiếp được.

d) Khi M di chuyển trên cạnh AB thì điểm E di chuyển trên đường cố định nào. Vì sao?

A E
M N

D D'
O H B

a) Tứ giác ABCD là hình thoi. Tính S ABCD .

OA OH OH R 1 R
cos 
AOB       OH 
Ta có OB OA R 2R 2 2

3R R 3R
 DH  OD  OH  BH  OB  OH  2 R  
2 và 2 2  DH  HB

Mặt khác HA  HC và BD  AC ( tính chất hai tiếp tuyến cắt nhau)

Vậy tứ giác ABCD là hình thoi ( tứ giác có hai đường chéo vuông góc với nhau tại mỗi đường và vuông góc
với nhau)

1 R 3 3 3R 2
S ABCD  AC.BD  AH .BD  .3R 
2 2 2 ( đvdt)
2
b) AB  AM .CN .

  0  
Xét ADM và CND có: MAD  DCN  120 ; ADM  CND ( so le trong)
63
Nhóm file Word toán THCS
 ADM ∽ CND (g.g)

AD AM
   AM .CN  AD.CD  AB 2
CN CD ( do AD  CD  AB )

c) Tứ giác AEBC nội tiếp

AC CN
AM .CN  AB 2  AC 2  
Ta có AM CA


MAC 
ACN  600  MAC ∽ ACN c.g .c 

 
AMC  CAN

  
Ta lại có: AMC  MEA  MAN ( góc ngoài tại đỉnh M của AME )

  CAB
CAN   MAN


MEA   600  CEA
 CAB   ABC
  600 , E , B
Suy ra: là hai đỉnh liên tiếp của tứ giác AEBC

 Tứ giác AEBC nội tiếp

d) E chuyển động trên đường cố định nào

 0
Vì AEC  60 (cmt) nên điểm E thuộc cung chứa góc 60 dựng trên đoạn AC .
0

Do M chỉ di chuyển trên AB nên E chỉ di động trên cung AB của đường tròn ngoại tiếp tam giác ABC.

Bài 45. Cho đường tròn


O; R  đường kính BC , điểm A ở bên ngoài đường tròn với OA  2R. Vẽ hai tiếp
tuyến AD, AE với đường tròn
O  ( D, E là các tiếp điểm).
a) Chứng minh tứ giác ADOE nội tiếp và xác định tâm I của đường tròn này

b) Chứng minh rằng tam giác ADE đều

c) Vẽ DH vuông góc với CE ( H thuộc CE ). Gọi P là trung điểm của DH ; CP cắt đường tròn
O  tại
Q . AQ cắt đường tròn O  tại M . Chứng minh AQ. AM  3R 2 .

d) Chứng minh đường thẳng AO là tiếp tuyến của đường tròn ngoại tiếp tam giác ADQ .

Hướng dẫn giải

64
Nhóm file Word toán THCS

a) Tứ giác ADOE nội tiếp D M

Do AD, AE là tiếp tuyến của


O  nên Q P C
I
AD  OD; AE  OE và AD  AE A K O H
B
 
Ta có: ADO và AEO cùng nhìn OA dưới một góc
vuông
E
nên tứ giác ADOE nội tiếp đường tròn đường kính
OA  2 R ,

tâm I là giao điểm của OA với cung DE của đường tròn


O 
b) ADE đều

 OD R 1  
cosDOA     DOA  600  AED  600
OA 2 R 2

AD  AE cmt   ADE
Mà đều
2
c) AQ. AM  3R

  
Xét ADQ và AMD có: MAD chung, ADQ  AMD ( góc tạo bởi tia tiếp tuyến và dây, góc nội tiếp cùng

chắn DQ )
AD AQ
 ADQ ∽ AMD  g .g     AD 2  AM . AQ
AM AD

 OD 3
sin DOA  Sin 600   AD  Sin 600.OA  .2 R  R 3
Mà OA 2

 
2
AM . AQ  AD 2  R 3  3R 2 .
Suy ra:

d) Đường thẳng OA là tiếp tuyến của đường tròn ngoại tiếp ADQ
Gọi K là giao điểm của OA và DE  K trung điểm DE
PD  PH  gt   KP
Mà là đường trung bình của DEH
 KP / / EH và KP  DH ( vì DH  EH )
 
Ta có QKP  QCE ( đồng vị)

65
Nhóm file Word toán THCS
  
QCE  QDE ( hai góc nội tiếp cùng chắn QE )

 QPK 
 QDE

Mà D, P là hai đỉnh liên tiếp của tứ giác DQKP


 Tứ giác DQKP nội tiếp ( cùng thuộc một cung chứa góc)
  0
Vậy DQK  DPK  90
  
Ta có: AKQ  QDE ( vì cùng phụ với DKQ )

QDE 
 AEQ 
( góc nội tiếp, góc tạo bởi tiếp tuyến và dây cùng chắn QE )

  ,K
AKQ  AEQ và E là hai đỉnh liên tiếp của tứ giác AEKQ
 Tứ giác AEKQ nội tiếp ( cùng thuộc một cung chứa góc)
  
Vậy QAO  QED ( hai góc nội tiếp cùng chắn QK )

QED  
ADQ ( góc nội tiếp, góc tạo bởi tia tiếp tuyến và dây cung cùng chắn QD )

 
Suy ra: QAO  ADQ

Do đó AO là tiếp tuyến của đường tròn ngoại tiếp ADQ

Thật vậy, dựng đường tròn


 I  ngoại tiếp ADQ , kẻ đường kính AA '
 ' A  QDA
  A'
 QA ( hai góc nội tiếp cùng chắn QA ) I
D
 
Mà QDA  QAO (cmt)
A
 ' A  QAO
 QA  Q
     0
Vậy A ' AO  A ' AQ  QAO  A ' AQ  QA ' A  90
 0
( do A ' QA  90  góc nội tiếp chắn nửa đường tròn)
 OA  IA , IA là bán kính đường tròn ngoại tiếp ADQ

 AO là tiếp tuyến của  I 

66
Nhóm file Word toán THCS

Bài 46. Từ điểm A ở ngoài đường tròn


O; R  , dựng các tiếp tuyến AB , AC và cát tuyến ADE ( D, E

thuộc
O  và D nằm giữa A, E ). Đường thẳng qua D vuông góc với OB cắt BC , BE lần lượt tại H và K .
Vẽ OI vuông góc với AE tại I .
a) Chứng minh rằng: tứ giác OIBC nội tiếp được đường tròn

b) Chứng minh rằng IA là tia phân giác BIC
2 1 1
 2

c) Gọi S là giao điểm của BC và AD . Chứng minh rằng: AD. AE  AC và AS AD AE
d) Chứng minh rằng tứ giác IHDC nội tiếp được đường tròn và DH  HK .
Hướng dẫn giải
a) Tứ giác OIBC nội tiếp
  
Ta có: OBA  AIO  ACO  90  B, I , C  đường tròn đường kính OA  Tứ giác OIBC nội tiếp.
0

B
K
F E
H I
D
S
O
A


b) IA là phân giác BIC .
Do AB, AC là tiếp tuyến của (O)  AB  AC
Xét trong đường tròn (OIBAC) có hai dây AB, AC sao cho AB  AC
 AB  AC
  AIC
 BIA 
( hai góc nội tiếp chắn hai cung bằng nhau)

 AI là tia phân giác góc BIC
2 1 1
2
 
c) AD.AE  AC và AS AD AE .

67
Nhóm file Word toán THCS
  
Xét ACD và AEC có EAC chung, ACD  AEC (góc nội tiếp, góc tạo bởi tia tiếp tuyến và dây cung chắn

DC )
AC AD
 ACD #  AEC (g.g)    AC 2  AD.AE
AE AC
   
Gọi F là giáo điểm của DK và OB, ta có DFO  DIO  90  Tứ giác DFIO nội tiếp đường tròn  HDI  BOI

(hai góc nội tiếp cùng chắn IF )
  
Xét đường tròn (OIBC) có: BOI  BCI (hai góc nội tiếp cùng chắn BI
  BCI
HDI  
tứ giác IHDC nội tiếp (cùng thuộc cung chứa góc)
  
Xét ACS và AIC có: IAC chung, ACS  AIC (góc nội tiếp cùng chắn hai cung bằng nhau)
 ACS # AIC  AC 2 = AS . AI
AD  AE  (AI  DI)  (AI  IE)  2AI (do DI  IE ) và AD.AE  AS.AI (=AC2 )
1 1 AE  AD 2AI 2
   
Suy ra: AD AE AD.AE AS.AI AS
d) DH  HK .
  
Tứ giác IHDC nội tiếp (cmt)  DCB  DIH (hai góc nội tiếp cùng chắn DH )
  
Mà DCB  BED (hai góc nội tiếp cùng chắn BD trong (O))
  BED
 DIH 
, mà hai góc ở vị trí đồng vị  HI // BE.
DKE có IH // KE, ID  IE(cmt)  HD  HK.
Bài 47. Cho đường tròn (O; R), đường thẳng d không qua O cắt đường tròn tại hai điểm A, B. Từ một điểm C
trên d (C nằm ngoài đường tròn (O) và A nằm giữa B, C), kẻ hai tiếp tuyến CM, CN với đường tròn (O)
(M, N  (O)) . Gọi H là trung điểm của AB , tia HO cắt tia CN tại K.
a) Chứng minh bốn điểm C, O, H, N thuộc môt đường tròn.
b) Chứng minh: MN.KC  KH.KO.
c) Đoạn thằng CO cắt (O) tại I, chứng minh I cách đều CM, CN và MN.
d) Một đường thẳng đi qua O và song song với MN cắt các tia CM, CN lần lượt tại E và F. Xác định vị trí
của C trên d sao cho diện tích tam giác CEF đạt giá trị nhỏ nhât.
Hướng dẩn giải

68
Nhóm file Word toán THCS
a) C, O, H, N thuộc môt đường tròn E
M
Đường kính chứa OH đi qua trung điểm H
B
của dây AB không qua tâm  OH  AB . H
  A
Ta có: OHC  ONC  90
 O, H, C, N thuộc đường tròn đường kính C d
OC. O I
b) MN.KC  KH.KO.
 KN HK
cosHKC  
KO KC
 KN.KC  KH.KO . N
F
K

c) Điểm I cách đều CM, CN và MN.


 
Do OM = OI = R  OMI cân tại O  OMI  OIM
Ta lại có OC là đường trung trực của MN nên OC  MN.
     
Suy ra: NMI  OIM  90  CMI  OMI  NMI  CMI
 
Vậy MI là tia phân giác NMC ; mà CI là tia phân giác NCM (tính chất hai tiếp tuyến cắt nhau)  I là tâm
đường tròn nội tiếp CMN  Điểm I cách đều CM, CN và MN.
S
d) Định vị trí của C trên d để CEF nhỏ nhất
Do MN // EF  CMN # CEF mà CMN cân tại C  CEF cân tại C nên đường cao CO cũng là đường
trung tuyến.
1 1
SCEF  OC.EF= OC.(2OE)=OC.OE = OM.CE = OM.(ME + CM)
2 2
 2OM. ME.CM  2OM. OM 2  2R 2 .
Dấu đẳng thức xảy ra  ME  CM  OCE vuông cân tại O
 OMC vuông cân tại M  OC  OM 2  R 2

Vậy
minSCEF  2R 2 .

Bài 48. Cho đường tròn (O; R), đường kính AB, trên AB lấy hai điểm I, J đối xứng nhau qua O (I  OA); M
là điểm bất kỳ trên đường tròn (O) (M  A; M  B). Các đường thẳng MI, MO, MJ cắt đường tròn (O) lần
lượt tại các điểm C, E và D. Đường thẳng CD cắt đường thẳng AB tại F.
2 2
a) Chứng minh rằng: FD.FC = FB.FA = OF - R .
b) Qua D kẻ đường thẳng song song với AB cắt ME ở L. Dựng OH vuông góc với CD tại H. Chứng minh
rằng: HL // CM .
c) Chứng minh EF là tiếp tuyến của đường tròn (O) .
Hướng dẩn giải

69
Nhóm file Word toán THCS
2 2
a) FD.FC = FB.FA = OF - R . C
 E
Xét FDB và FAC có CFA góc chung, H
  FAC
FDB  
(cùng bù với CDB
 FDB # FAC (g.g) . N D
FD FB L
   FD.FC  FA.FB A
FA FC I O J B F
 (OF  OB)(OF  OA)
 (OF  R)(OF  R)  OF2  R 2
M

b) HL // CM .
Kéo dài DL cắt CM tại N.
Trong đường tròn (O) : OH  CD tại H  HC  HD (định lí đường kính và dây cung).
Vì DN // IJ, áp dụng hệ quả của định lí Ta – Lét có:
OJ OM OI
  ,
LD ML NL mà OI  OJ (gt)  DL = NL.
Vậy HL là đường trung bình trong CND  HL // CM.
c) EF là tiếp tuyến của đường tròn (O) .
    
Ta có: MCD  LHD (đồng vị, HL // CM), MCD  DEL (hai góc nội tiếp cùng chắn MD )
 
 LHD  DEL, mà H, E là hai đỉnh liên tiếp của tứ giác
 tứ giác HLDE nội tiếp (cùng thuộc một cung chứa góc).

 HEL  
 HDL (hai góc nội tiếp cùng chắn HL ).
   
Mặt khác HDL  HFO (đồng vị)  HEL  HFO, E và F là hai đỉnh liên tiếp của tứ giác
 tứ giác HEFO nội tiếp.
  OHF
 OEF   90,
nghĩa là EF  OE tại E, OE = R nên EF là tiếp tuyến của đường tròn (O; R)
Bài 49. Cho tam giác ABC có ba góc nhọn nội tiếp trong đường tròn (O; R) có đường cao AH. Các tiếp tuyến
của (O) tại B và C cắt nhau tại D. Gọi I và K lần lượt là hình chiếu của A trên DB và DC.
a) Chứng minh các tứ giác AHBI, AHCK nội tiếp được.
b) Chứng minh AHI và AKH đồng dạng.
c) Gọi M, N lần lượt là trung điểm của AI, AK. tam giác ABC phải thỏa mãn điều gì để AH  AM  AN.
d) Giả sử AB  R 2, BC = R 3 (C nằm cùng phía với A só với bờ là OB). Tính AH theo R.
Hướng dẩn giải

70
Nhóm file Word toán THCS
I B
B

M
D O A
O
H
A C
C
N

a) Tứ giác AHBI, AHCK nội tiếp


 
Ta có: AHC  AKC  180  Tứ giác AHCK nội tiếp.

AHC 
 AKC  180  Tứ giác AHBI nội tiếp.
b) AHI # AKH
  
Ta có: HBA  HIA (hai góc nội tiếp cùng chắn AH ).

AHK 
 ACK 
(hai góc nội tiếp cùng chắn AK ).

ACK 
 HBA 
(góc tạo bởi tia tiếp tuyến và dây, góc nội tiếp cùng chắn AC ).
  AHK.
 AIH 
 
Lý luận tương tự: IHA  AKH. Suy ra AHI # AKH (g.g)
c) ABC phải thỏa mãn điều gì để AH  AM  AN.
AH AI
AHI # AKH    AH 2  AI.AK
Do AK AH
Mà AI  2AM; AK  2AN nên AH  AM.AN  AH  2 AM.AN.
2

Ta có: AM  AN  2 AM.AN  AH.


Dấu đẳng thức xảy ra khi và chỉ khi AM  AN.

 AI  AK  A thuộc tia phân giác của góc BDC  ABC cân tại A.
d) Tính AH
Trong đương tròn (O; R) :

AB  R 2  AOB  90 (AB là cạnh hình vuông nội tiếp (O; R)).

BC  R 3  BOC  120 (BC là cạnh tam giác đều nội tiếp (O; R)).
 1 
BCA  sñAB  45  AHC
Suy ra: 2 vuông cân tại H
Trang 82-83-84-85-86
Đặt AH  CH  x; HB  y , do AB  AC  0  x  y.

71
Nhóm file Word toán THCS
x  y  R 3
 x  y   2 xy  2 R
2 2 2 2
 x  y  2 R
2

   R2
 x  y  R 3  x  y  R 3  xy 
Ta có :  2
Vậy x, y là nghiệm của phương trình :
2 R2
t  R 3t  0
2
R2
 
2
  R 3  4.  R 2    R.
2

t1 
R 3R R

 3 1 
2 2

t2 
R 3R R

 3 1 
2 2

Do x  y  x 
R  3 1 .
2

AH 
R  3 1 .
Vậy 2
Bài 50. Cho đường tròn (O), BC là một dây cung của (O), ( BC  2 R ). Các tiếp tuyến với đường tròn (O) tại B
và C cắt nhau ở A. Gọi M là điểm thuộc cung nhỏ BC, hình chiếu của M lần lượt trên AB, BC, AC là K, I, H.
MB cắt IK tại E, MC cắt IH tại F.
a) Chứng minh các tứ giác BIMK, CIMH nội tiếp.
2
b) Chứng minh : MI  MH .MK .
c) Chứng minh : EF  MI .
d) Gọi giao điểm thứ hai của hai đường tròn (MEK) và (MFH) là N. Chứng minh MN luôn đi qua một điểm cố
định.
Hướng dẫn giải

a) Tứ giác BIMK; CIMH nội tiếp.


  0
Ta có : MKB  MIB  180  Tứ giác BIMK nội tiếp.

MHC   1800 
 MIC Tứ giác CIMH nội tiếp.

72
Nhóm file Word toán THCS
MI 2  MH .MK
b)
  
Ta có : MIK  MBK ( hai góc nội tiếp cùng chắn cung MK )
  
MBK  BCM ( góc tạo bởi tia tiếp tuyến và dây, góc nội tiếp cùng chắn MB ).
  
BCM  MHI ( hai góc nội tiếp cùng chắn cung MI )
  MHI
 MIK 
.
  MIH
MKI   MIK ∽ MHI  g .g 
Tương tự : .
MI MK
  MI 2  MH .MK .
Do đó : MH MI
EF  MI
c)
Dựa vào kết quả câu b, ta được :

EMF   EMF
 EIF   M
 EIM  IF  EMF
 
 MCB 
 MBC  1800
 Tứ giác EMFI nội tiếp
  EF
 EIM  M 
( hai góc nội tiếp cùng chắn ME )
 
EIM  MBC  cmt 


 EFM  ,
 MBC mà hai góc ở vị trí đồng vị nên EF // BC
MI  BC  gt   EF  MI .
Ta lại có
d) MN luôn đi qua một điểm cố định.
 
 EF  MBI  dong vi, EF / / BC 
M

Ta có :


 MBI  MKE hai goc noi tiep cung chan MI



 MKE M EF 
EF là tiếp tuyến của đường tròn ngoại tiếp KME.
Thật vậy :
Kẻ đường kính ED của đường tròn ngoại tiếp KME.

 MDE



 MKE  
hai goc noi tiep cung chan ME 

 MKE M EF  E  cmt 
Ta có :  1


 MDE E
1

DME  900 ( góc nội tiếp chắn nửa đường tròn ).
     0
Do đó : DEF  E1  E2  MDE  E2  90
 EF  DE , DE là đường kính của đường tròn  KME  .
 EF là tiếp tuyến của đường tròn ngoại tiếp KME.
Lý luận tương tự, ta được: EF là tiếp tuyến của đường tròn ngoại tiếp MFH .
Gọi S, J lần lượt là giao điểm của MN với EF , BC.
Ta có :

73
Nhóm file Word toán THCS
SE SM
SEM SNE  g .g     SE 2  SM .SN
SN SE
SF SM
SFM SNF  g .g     SF 2  SM .SN
SN SF
2 2
Suy ra : SE  SF  SE  SF .
Áp dụng hệ quả Ta – lét, có :
SE MS SF
   BJ  JC 
BJ MJ JC J cố định.
Vậy MN luôn luôn đi qua trung điểm J của BC.

Bài 51. Cho đường tròn (O; R), BC là một dây cung của đường tròn (O)
 BC  2 R  , các tiếp tuyến với
 0
đường tròn ( O) tại B và C cắt nhau tại A sao cho BAC  60 . Gọi M là điểm bất kỳ thuộc cung nhỏ BC,
tiếp tuyến với đường tròn (O) tại M cắt AB, AC lần lượt theo thứ tự ở D, E.
a) Tính chu vi tam giác ADE theo R.
b) Gọi giao điểm của OD, OE lần lượt là I và K. Chứng minh các đường thẳng OM, DK, EI đồng quy.
c) Chứng minh : DE =2.IK.
3R 2
 0 CP.BS  .
d) Trên AC, BC, AB lần lượt lấy các điểm P, Q, S sao cho PQS  60 . Chứng minh : 4
Hướng dẫn giải:

ADE
a) Tính chu vi theo R.
AB, AC là các tiếp tuyến của (O) (gt).
Áp dụng tính chất hai tiếp tuyến cắt nhau, ta có
AB  AC ; DM  DB; ME  MC. Do BAC   60 0  BOC
  1200
 BC  R 3 ( BC là cạnh của tam giác đều nội tiếp (O;R)).
Ta có :
AD  DE  AE  AD  MD  ME  AE   AD  DB    EC  AE   AB  AC  2 AB  2 BC  2 R 3
b) OM, DK, EI đồng quy

BOM 
MOC 
BOM 
 MOC 
BOC

DOM  
; MOE  
 DOE 
 DOM 
 MOE    600.
Ta có : 2 2 2 2
 
Vậy DOK và DBK cùng nhìn DK dưới 1 góc 600, O và B là hai đỉnh liên tiếp của tứ giác BOKD  Tứ
giác BOKD nội tiếp.

 DKO 
 1800  DBO  1800  900  DK  OE.
74
Nhóm file Word toán THCS
 0
Lý luận tương tự : tứ giác ECOI nội tiếp  OIE  90  EI  OD.
Mà MO  DE ( vì DE là tiếp tuyến của (O) )
 OM, DK, MI là ba đường cao của ODE nê đồng quy.
c) DE = 2IK.
 
Ta có : DIE và DKE cùng nhìn DE dưới một góc vuông
 Tứ giác IKED nội tiếp  OIK   OED .
IK OI 1
OIK ∽ OED  g.g      DE  2 IK .
Vậy ED OE 2
1
I  900 ; IOE
  600 OI  OE
( Vì OIE có nên 2 ).
3R 2
CP.BS 
4
d) .
 
 PQB  PQS  Q  600  Q 
1 1  P
Q 
    1 1

Ta có :  PQB  ACB  P1  60 0
 P1  goc ngoai dinh Q cua CQP 
  0
Mà ACB  ABC  60 nên
CP CQ
CPQ ∽ BQS  g.g   
BQ BS
 CP.BS  BQ.CQ  BQ  BC  BQ     BQ 2  BC .BQ 
2
 BC BC 2  BC 2  BC  BC 2 BC 2
   BQ 2  2 .BQ       BQ     .
 2 4  4  2  4 4
BC
BQ   QC  QB.
Dấu đẳng thức xảy ra khi và chỉ khi 2

Bài 52. Cho tam giác ABC có ba góc nhọn nội tiếp đường tròn (O), đường phân giác trong của góc BAC
 0
cắt cạnh BC tại L và cắt đường tròn (O) tại N ( giả sử ABN  90 ). Từ L ta hạ các đường vuông góc LK và
LM theo thứ tự xuống các cạnh AB và AC.
a) Chứng minh ABL ANC.
2
b) Chứng minh rằng : AL  AB. AC  BL.CL.
BL  b, CL  a  a  b  .
c) Tiếp tuyến tại A của đường tròn (O) cắt đường thẳng BC tại E. Biết Tính AE
theo a và b.
S  S ABC .
d) Chứng minh rằng : AKNM
Hướng dẫn giải

75
Nhóm file Word toán THCS

ABL ∽ ANC
a) .
  NAC
BAN   gt  .
Xét ABL và ANC có :

ABC   ANC ( hai góc nội tiếp cùng chắn cung 
AC ).
ABL ∽ ANC  g .g  .
Do đó
AL2  AB. AC  BL.CL
b) .
Ta có :
AL2  AB. AC  BL.CL
 AL2  BL.CL  AB. AC
 AL2  AL.LN  AB. AC


AL (AL+LN)= AB.AC


AL.AL = AB.AC (đẳng thức đúng)

AB AL
   AB. AC  AN . AL
  AN AC
Vì ABL ANC

BL AL
ABL CNL  g .g   NL  CL  BL.CL  AN .NL

c) Tính AE theo a và b

ELA
2

  1 sđ AB  sđ NC


Ta có : (góc có đỉnh ở bên trong đường tròn)


1
2

sđ AB  sđ NB


(N là điểm chính giữa của BC)

76
Nhóm file Word toán THCS
1  
 sđ AN  EAN AN
2
(Góc tạo bởi tia tiếp tuyến và dây cung cùng chắn )

 EAL 
 cân tại E EA = EL.

EA EB
   EA2  EC.EB
EAB ECA EC EA
Mặt khác (g.g)

 EA2   EL  BL  EL  LC   EA2   EA  b  EA  a 

 EA2  EA2   a  b  EA  ab  EA   EA  b  EA  a 

ab
 EA2  EA2   a  b  EA  ab  EA  a  b 
a b

d) SAKNM = SABC


ABN  
ACN  1800 
ABN  900  
ACN  900 
do tứ giác ABNC nội tiếp nên mà . Do đó nếu hạ NH AB
và NI AC thì H ở ngoài đoạn AB (có thể trùng với B) và I nằm giữa A và C (có thể trùng C).

Ta có: LM // NI (cùng vuông góc với AC)

 S LMN  S LMI
(1)

Tương tự : NH // LK (vì cùng vuông góc với AB)

 S LNK  S LHK
(2)

Từ (1) và (2) cho:

 S LMN  S LNK  S LMI  S LHK  S LMC  S LIC  S LBK  S LBH


(3)

NBH NIC
và có: NB = NC (hai cung bằng nhau nên hai dây bằng nhau)

  I  900
H
NH = NI (tính chất tia phân giác ),

 NBH  NIC
(cạnh huyền - cạnh góc vuông)
77
Nhóm file Word toán THCS
 BH = IC, mà LK = LM (tính chất tia phân giác)

 S LBH  S LIC
(4)

S LMN  S LNK  S LBK  S LMC


Từ (3), (4) ta được:

S AKNM  S ABC
Do đó: .

Bài 53. Cho tam giác ABC (AB< AC), đường trung tuyến AM. Đường tròn (O 1) nội tiếp tam giác ABM tiếp
xúc với các cạnh BM, MA, AB lần lượt tại D, E, F. Đường tròn (O 2) nội tiếp tam giác ACM tiếp xúc với các
cạnh CM, MA, AC lần lượt tại I, J, K. Chứng minh:

a) MO2 // DE và MO1 // IJ

b) MI.MD=O1D.O2I.

AB  AM  BM AC  AB
AE  EJ 
2 2
c) và

d) Các đường thẳng IJ , DE, O1O2 cắt nhau tại một điểm.

Hướng dẫn giải

E K
F
O1 O
O2
H J N
B C
D M I

a) MO2//DE và MO1 //IJ.

Áp dụng tính chất hai tiếp tuyến cắt nhau, ta có :


 
DE O1M, MO2 O1M (hai tia phân giác của hai góc kề bù)

78
Nhóm file Word toán THCS
 MO2//DE
Lí luận tương tự : MO1//IJ

b) MI.MD=O1D.O2I

DMO1 IO2 M   I  900 DMO


D   
1 1  MO2 I IMO2
Xét và có : ; (cùng phụ với )

 DMO1 IO2 M
(g.g)

O1 D MD
  IM .MD  O1D.IO2
 IM IO2
.

AB  AM  BM AC  AB
AE  EJ 
2 2
c) và

Áp dụng tính chất hai tiếp tuyến cắt nhau , ta có: AE= AF, BF = BD và EM = MD
 AE  AM  EM  AM  MD

AF = AB -BF = AB -BD
Vậy :

AE + AF = AB +AM – (MD+BD)
AB  AM  BM
AE 
 2
2AE = AB +AM – BM =>
AM  AC  MC
AJ 
2
Lí luận tương tự , ta được :

1
EF  AJ  AE  AM  AC  MC  ( AB  AM  MB )
2
Suy ra :

1
 AC  AB
2
(do MB = MC- (gt))

AC  AB

2
(do AC> AB)

79
Nhóm file Word toán THCS
d) IJ , DE, O1O2 cắt nhau tại một điểm

Gọi O là giao điểm của DE và IJ, ta cần chứng minh O1 ; O ; O2 thẳng hàng .

Gọi H là giao điểm của O1M và DE

N là giao điểm của O2M và IJ

MD O1M

DMO1 IO2 M  OI 2 O2 M
(g.g)

O1M 2 MD2 MH .O1 M OM MH ON


  2
  1  
O2 M 2
O2 I O2 N .O2 M O2 M O2 N O2 N

 O1MO2 ONO2 O M  O
O O N
1 2 2

(c.g.c). Vậy

Do đó O1; O; O2 thẳng hàng, nghĩa là các đường thẳng IJ, DE và O1O2 cắt nhau tại một điểm .

Bài 54. Cho tam giác ABC vuông tại A, đường cao AH. Gọi (P), (Q) theo thứ tự là đường trong nội tiếp hai tam
giác AHB và AHC. Kẻ tiếp tuyến chung ngoài (khác BC) của hai đường tròn (P), (Q) nó cắt AB, AH , AC theo
thứ tự ở M, K ,N.

HPQ ABC
a) Chứng minh rằng : và KP//AB

b) Chứng minh tứ giác BMNC nội tiếp và năm điểm A, M, P, Q, N thuộc một đường tròn.

AED
c) Gọi D, E theo thứ tự là giao điểm của PQ với AB và AC. Chứng minh vuông cân.

d) Cho BC cố định, A là điểm di động trên nửa đường tròn đường kính BC. Tìm vị trí của A để PQ đạt độ dài
lớn nhất.

Hướng dẫn giải

80
Nhóm file Word toán THCS
A

1 2 3
N

K
M
P Q
D

B C
H

HPQ ABC
a) và KP//AB.

ABH CHA
Do P, Q là tâm đường tròn nội tiếp và nên:

 
A  HAC 
HBP 
ABC
2
2 2

0
 90
AHP  
AHQ   450
2


HAC 
ABC
Mà (Cùng phụ với góc BAH)


HAC 
ABC
Mà (Cùng phụ với góc BAH)

 
A  HAC 
HBP 
ABC
2
2 2

0
 90
AHP  
AHQ   450
2


HAC 
ABC
Mà (Cùng phụ với góc BAH)

A  HBP

2

81
Nhóm file Word toán THCS
 BHQ AHQ
(g.g)

HP AB
 
HQ AC

  0  
HBA HAC AHB  AHC  90 HBA  HAC
Ta lại có : và : ; (cmt)

BH AB
 
 HBA HAC AH AC
(g.g)

HP AB
   PHQ

HQ AC BAC  900  HPQ ABC
Do đó : , mà (c.g.c)

*KP//AB?

MKH 
NKH
KP, KQ là hai tia phân giác của hai góc kề bù và (Tính chất hai tiếp tuyến cắt nhau) =>

PKQ  900 
PHQ  900
, mà

  PHQ
PKQ   1800
 => Tứ giác PKQH nội tiếp


HQP 
 HKP
 (1)


HQP 
C
Từ kết quả trên: cho

 
C A1 
HAC
Mà (cùng phụ với )


HQP 
A1
 (2)

82
Nhóm file Word toán THCS
 
A1  HKP
Từ (1) và (2) cho: , mà hai góc này ở vị trí đồng vị nên KP//AB. Lý luận tương tự ta được:
KQ//AC

b) Tứ giác BMNC nội tiếp và năm điểm A, M, P, Q, N thuộc một đường tròn.

 
C 
A1  HKP 
 MKP
Dựa vào kết quả câu trên ta có:


MKP 
AMN  
C AMN
Mà (so le trong, KP//AB)=>

Suy ra tứ giác BMNC nội tiếp (có góc trong bằng góc ngoài tại đỉnh đối diện).


A1  
AMN 
C
Ta có: (cùng bằng )

 KMA
cân tại K=> KM = KA.

Tương tự: KN = KA

A  
AQK AKQ
2

Ta lại có: (vì cùng bằng góc A3)=> cân tại K

 KQ = KA

Tương tự : KP = KA.

Do đó: KM = KA= KN= KQ = KP => năm điểm A,M,P,Q,N cùng thuộc một đường tròn

AED
c) vuông cân.


HQP 
C
Từ câu a, ta có: => Tứ giác HQEC nội tiếp (có góc trong bằng góc đối ngoài), do đó:
  QHA
QEA 

= 450
A  900 , E
  450
AED AED
có nên vuông cân tại A.

83
Nhóm file Word toán THCS
d) PQ đạt độ dài lớn nhất.

ABH ; ACH ABC


Gọi (P; r1); (Q; r2)lần lượt là đường tròn nội tiếp . (O;r) là đường tròn nội tiếp

BCA BAH ACH


Ta chứng minh được .

r1 AH r AH
   2 
r AC r AB
;

r12  r22  1 1  1
 2
 AH 2  2
 2 
 AH 2 . 1
r  AC AB  AH 2

 r 2  r12  r22

PHQ  PQ 2  PH 2  HQ 2  2  r12  r22   2r 2


Mặt khác vuông tại H

Suy ra : PQ lớn nhất khi và chỉ khi r lớn nhất.

OBC

BOC 
  OCB
 1800  OBC   1350 

Vậy O thuộc cung chứa góc 1350 dựng trên đoạn BC thuộc nửa mặt phẳng bờ BC cùng phía với nửa đường

tròn đuường kính BC.

Dễ thấy O là điểm chính giữa cung đó thì r lớn nhất

Lúc đó, A là điểm chính giữa của nưả đường tròn đườngkính BC

84
Nhóm file Word toán THCS
A

r r2
r1

Bài 55: Cho tam giác ABC nhọn nội tiếp đường tròn
O; R  . Các tiếp tuyến tại B và C với đường tròn O 
cắt nhau tại E , AE cắt đường tròn
O  tại D (khác điểm A ).
a) Chứng minh rằng tứ giác OBEC nội tiếp.

b) Từ E kẻ đường thẳng d song song với tiếp tuyến tại A của đường tròn
O  , d cắt các đường thẳng
AB, AC lần lượt tại P, Q . Chứng minh: AB. AP  AD.AE .

 
c) Gọi M là trung điểm đoạn thẳng BC . Chứng minh: EP  EQ và PAE  MAC .

BC 2
AM .MD 
d) Chứng minh rằng: 4 .

Hướng dẫn giải

a) Tứ giác OBEC nội tiếp A



Do EB, EC là hai tiếp tuyến của
O  nên:
  OCE
OBE   900 ;
 tứ giác OBEC nội tiếp đường tròn đường kính

OE . O
b) AB . AP  AD . AE 1
Ta có: M
B   C d
 
 APE  BAx so le trong, d  Ax  2
  
 Q
 BAx  
BDA 
 góc tao boi tia tiep tuyen và dây, g óc noi tiep cùng chan 
AB D N

 
APE  BDA . Vậy 
E
AD AB
ADB ∽ APE  g .g     AB. AP  AD. AE
AP AE . P

EP  EQ   MAC
PAE 
c) và .

85
Nhóm file Word toán THCS
    
Ta có: BAx  B1 (cùng chắn AB ) , B1  B2 ( đối đỉnh);
 
BAx APE  cmt 
.
   BEP
APE  B
Suy ra: 2 cân tại E  BE  PE .

Tương tự: ECQ cân tại E  CE  EQ .

Ta lại có: EB  EC ( tính chất hai tiếp tuyến cắt nhau).

Vậy: EP  EQ .
   
ABC và AQP có: PAQ chung; APQ  BAx  ACB

BC
AC BC MC
   2 
AP PQ PQ EP
  ABC ∽ AQP  g.g  2

AMC ∽ APE  c.g .c   MAC 
 PAE
Vậy .

BC 2
AM .MD 
4
d) .

Cách 1:
   
ACM và ADB có MAC  BAD  cmt  ; ADB  ACB (hai góc nội tiếp cùng chắn 
AB ).

AM CM BM
  
 ACM ∽ ADB  g.g  AB BD BD .

     
Ta lại có: BAM  BAD  MAD  MAD  MAC  DAC ;

DAC 
 DBM 
(hai góc nội tiếp cùng chắn DC ).
 
Suy ra: BAM  DBM .
AM BM
  
MAB và MBD có BAM  DBM (cmt); AB BD (cmt)

MB MA 2 BC 2
 MAB ∽ MBD  c.g.c     MA.MD  MB 
MD MB 4 .

Cách 2:
86
Nhóm file Word toán THCS

Gọi N là giao điểm của tia AM với


O  .
MB MA BC 2
BAM ∽ NMC  g.g     MA.MN  MB.MC 
MN MC 4 (1)

BAD   cmt   BD
 NAC   CN

Do ( hai góc nội tiếp bằng nhau nên hai cung chắn bằng nhau).

 BD  CD (hai cung chắn bằng nhau nên hai dây bằng nhau) và DBM 
 MCN .

MBD  MCN  g .c.g   MD  MN


Vậy (2)

BC 2
 MA.MN  MA.MD
Từ (1) và (2) cho 4

Bài 56: Gọi M , N lần lượt là trung điểm các cạnh AB, CD của hình chữ nhật ABCD . Biết rằng đường tròn

ngoại tiếp hình chữ nhật ABCD có đường kính bằng 8  2 3 và tồn tại điểm I thuộc đoạn MN sao cho
  450 ; IDA
DAI   300
.
ABCD
a) Tính diện tích hình chữ nhật .
H,K AID BIC P Q
b) Gọi lần lượt là trọng tâm của các tam giác và . Gọi và lần lượt là trung điểm của
AD BC E F IN KH PQ
và . Gọi và lần lượt là giao điểm của với và . Tính diện tích tam giác
NKH
.

Hướng dẫn giải


S A M
a) Tính ABCD :      B
Đường tròn ngoại tiếp hình chữ nhật ABCD 45 0
a
có đường kính bằng 8  2 3 (gt)
I
Do đó: AC  BD  8  2 3 
  900  DAI
  450 K H
IAM  E 
  900   P  Q
IDN ADI  600 F
AB
AM   gt 
2
CD
DN   gt  300
2 .
AB  CD (tứ giác ABCD là hình chữ nhật)   C
D 
AB N
AM  DN 
Do đó: 2 .

87
Nhóm file Word toán THCS
  450
IMA vuông tại M có IAM nên IMA vuông cân tại M
AB
 MI  AM 
2 .

  600
NDI vuông tại N có IDN nên NDI là nửa tam giác đều.

AB 3
 IN  DN 3  ; AD  MN ; MN  MI  IN
2

AB AB 3 AB

2

2
 AD 
2
1  3  AD
.
 
ABD vuông tại A nên: AB 2  AD 2  BD 2

AB 2
 
2
AB 2  1 3  8 2 3
4

AB 2

4

4 1 2 3  3  8  2 3 
 AB 2  4  AB  2 .

2
Do đó:
AD 
2

1 3  1 3
.

S ABCD  AB. AD  2 1  3   (đvdt).
S NKH
b) Tính :

IK 2
 
Ta có AID có K là trọng tâm, IP là đường trung tuyến IP 3 .

IH 2

Tương tự: IQ 3.

IK IH  2 
  KH  PQ
IPQ có IP IQ  3 

IE KH IK 2 2
    KH  PQ
Do đó: IF PQ IP 3 3 .

Mà PQ  AB  2 .

88
Nhóm file Word toán THCS
4
KH 
Do đó: 3.

AB 3 AD 1 3 3 1
IF  IN  NF    3 
2 2 2 2 .

2 2 3 1 3 1
IE  IF  . 
3 3 2 3 .

3 1 2 3 1
NE  IN  IE  3  
3 3 .

S NKH 
1
NE.KH  

1  2 3 1  4 2 2 3 1 
. 
2 2  3  3 9
(đvdt).

 0
Bài 57: Cho hình vuông ABCD có độ dài cạnh bằng a . Một góc xAy  45 quay quanh A cắt BC tại N và
cắt CD tại M . Gọi P, Q lần lượt là giao điểm của BD với AM , AN .

a) Chứng minh rằng các điểm P, Q, M , N , C cùng thuộc một đường tròn.

b) Chứng minh rằng MN luôn tiếp xúc với một đường tròn cố định.
PQ 1

c) Chứng minh: MN 2.

d) Tìm giá trị nhỏ nhất của diện tích tam giác AMN theo a .

Hướng dẫn giải

a) P, Q, M , N , C cùng thuộc một đường tròn.


A
 B
Áp dụng tính chất đường chéo của hình vuông,
 0 Q
ta có: BDM  45 . 
  N
 QDM  MAQ  450 , A, D là hai đỉnh liên tiếp 
x
của tứ giác ADMQ .
 tứ giác ADMQ nội tiếp (cùng thuộc cung chứa góc) P  
H

 MQA  
ADM  MQN  900 .
D  
Lý luận tương tự: tứ giác ABNP nội tiếp M C

 MPN 
 900 mà MCN  900 y
 P, Q, M , N , C cùng thuộc một đường tròn đường kính MN .
MN
b) luôn tiếp xúc với một đường tròn cố định.

89
Nhóm file Word toán THCS
  0
Kẻ AH  MN . Xét ADQ và CDQ có: AD  DC (cạnh hình vuông), DQ chung, ADQ  CDQ  45
 ADQ  CDQ  c.g .c   AQ  CQ
.
  900 , MAQ
AQM có Q   450  AQM vuông cân tại Q .


 AQ  QM  QM  QC  QCM cân tại Q  QMC 
 QCM (1)
 
Mặt khác: QMC  DAQ (do tứ giác ADMQ nội tiếp) (2)
 
QCM  QNM (do tứ giác QNCM nội tiếp) (3)
 
DAQ  BNA (so le trong, BC  AD ) (4)
 
Từ (1); (2); (3); (4) cho: BNA  ANH
  0  
AHN và ABN có: H  B  90 ; BNA  ANH  cmt  ; AN chung

 AHN  ABN (cạnh huyền - góc nhọn)  AH  AB (không đổi).

Vậy MN luôn tiếp xúc với đường tròn (A; AB) cố định.
PQ 1
=
MN 2
c)
  
 APQ và  AMN có: MAN chung, APQ  ANM (góc trong bằng góc ngoài tại đỉnh đối diện của tứ giác
MNPQ nội tiếp)   APQ ∽  AMN (g.g)
PQ AP AP 1
 = = =
MN AM AP. 2 2 (Vì  APN vuông cân tại P nên AN=AP. 2 ).
d) SAMN nhỏ nhất
AH.MN
SAMN = , AH = AB = a
2 (không đổi).
SAMN nhỏ nhất  MN nhỏ nhất.
MN = MH + HN = MD + NB (do MH = MD; NH = ND)
 MN + CM + CN = (MD + CM) + (CN + NB) = DC + BC = 2a.
Áp dụng bất đẳng thức B. C. S: (ax + by)2 ≤ (a2 + b2)(x2 + y2), ta có:

CM + CN ≤
2(CM 2 +CN 2 ) = 2MN 2 = 2MN
 2a = MN + CM + CN ≤ MN + 2MN
2a
= 2( 2-1).
 MN( 2 + 1) ≥ 2a  MN ≥ 2+1 a.
Dầu bằng xảy ra  CM = CN.
Vậy minSAMN = ( 2 - 1)a2.

90
Nhóm file Word toán THCS

Bài 58. Cho tam giác ABC (AB < AC) có ba góc nhọn nội tiếp đường tròn (O; R). Gọi M là điểm chính giữa
cung nhỏ BC, AM cắt BC tại D.


a) Chứng minh rằng AM là tia phân giác góc BAC .

b) Tiếp tuyến tại A với đường tròn (O) cắt đường thẳng BC tại S. Chứng minh:  SAD cân.

c) Giả sử AB = 4cm; BC = 5cm; AC = 6cm. Chứng minh:  SAB cân.

d) Gọi O1 là tâm đường tròn nội tiếp  ABC, đường thẳng vuông góc với BO1 tại B cắt tia AM tai O2. Biết góc
OO = 900
O1 2 và:

+ Chứng minh các điểm O, O1, B, O2, C cùng thuộc một đường tròn nhận M là tâm.

+ Giả sử BC cố định thì điểm A ở vị trí nào để diện tích  ABC có giá trị lớn nhất?

Tính giá trị lớn nhất đó theo R.

Hướng dẫn giải


A'
A
A

O1 O

O
K C
B H D
S
B
D C

O2


BAC
a) AM là tia phân giác
  
Do M là điểm chính giữa BC  MB = MC

 BAM 
 CAM (hai góc nội tiếp chắn hai cung bằng nhau)

 AM là tia phân giác của BAC .

b) SAD cân

91
Nhóm file Word toán THCS
 1
SDA  

Ta có: 2 (sđ AB + sđ MC ) (góc có đỉnh ở bên trong đường tròn (O))
1 1
    
= 2 (sđ AB + sđ MB ) = 2 sđ AM = SAM (góc tạo bởi tia tiếp tuyến và dây chắn AM ).
  SAD cân tại S.

c) SAB cân
  
 SAB và  SCA có: ASC chung, SAB  SCA (hệ quả góc tạo bởi tia tiếp tuyến và dây cung)   SAB
∽  SCA (g.g)
SA SB AB AB 4 2
= = = =
 SC SA AC mà AC 6 3
SA SB AB AB SB 4 SB 4
. = .  
 SC SA AC AC  SC 9  BC 5
4 4
BC .5
 SB = 5 = 5 = 4 (cm)  SB = AB
Vậy  SAB cân tại B.
d) O, O1, B, O2, C cùng thuộc một đường tròn

+ Vì O1 là tâm đường tròn ngoại tiếp  ABC  BO1 là tia phân giác của ABC
O B  BO1
Mà 2 (gt)  BO2 là tia phân giác ngoài tại đỉnh B của  ABC.

Mắt khác AM là tia phân giác trong của  ABC  O2 là tâm đường tròn bang tiếp trong của A của  ABC.
 CO2 là tia phân giác ngoài tại đỉnh C
 0
 O1CO 2 = 90 (góc tạo bởi hai tia phân giác của hai góc kề bù).
OBO  O CO = O OO  90 0
Ta có: 1 2 1 2 1 2  B, O, C thuộc đường tròn đường kính O1O2
 O, O1, B, O2, C cùng thuộc một đường tròn.
 M =
BO ABO1 + MAB
1 (góc ngoài tại đỉnh O1 của  ABO1)
 BC + MAC
O   
= 1 (vì BAM  CAM , chứng minh trên)
 BC + MBC
O    
= 1 ( MAC và MBC là hai góc nội tiếp cùng chắn MC )
 BM
O
= 1
  MBO1 cân tại M  MB = MO1
 M =
BO MBO 2
Mà 2 (cùng phụ với hai góc bằng nhau)
  MBO2 cân tại M  MB = MO2 = MO1  M là tâm đường tròn đi qua năm điểm O1, O, O2, B, C.
+ Ta lại có:  O1OO2 vuông tại O, có OM là đường trung tuyến ứng với cạnh huyền nên O1M = OM = OB =
R = MB.
  BOM đều nên BOM  = 600
Kẻ đường kính MA’ cắt BC tại K  OM  BC .

92
Nhóm file Word toán THCS
R 3
 2  BC = 2BK = R 3
BK = OB.sin BOK = R.cos600 =
R

OK = OB.sin BOK = R.cos600 = 2
1 R 3 R 3 R 3
Ta có: SABC = 2 BC.AH = 2 .AH ≤ 2 (OA + KO) = 2 (OA’ + OK)
R 3 3R 3 3R 2
= 2 . 2 = 4

Dấu “=” xảy ra  H  K và A  A’  A’ là điểm chính giữa cung lớn BC .
3 3R 2

Vậy maxSABC = 4 (Khi A’ là điểm chính giữa cung lớn BC nghĩa là  ABC đều nội tiếp (O; R)).

Bài 59: Cho tam giác ABC cân tại A nội tiếp đường tròn
O; R  . Điểm M thuộc cung AC ( M  A và
M  C ). Gọi Cx là tia qua M .

a) Chứng minh tia MA là tia phân giác của góc BMx .

b) Gọi D là điểm đối xứng của A qua O . Trên tia đối của tia MB lấy MH  MC . Chứng minh rằng
MD / / CH .

c) Gọi I và K theo thứ tự là trung điểm của CH và BC . Xác định tâm đường tròn đi qua bốn điểm A, I , C , K .

d) Khi M di động trên cung AC thì trung điểm E của HM chuyển động trên đường nào.

Hướng dẫn giải


a) Tia MA là tia phân giác của góc BMx
 ABC cân tại A  AB  AC
 AC   AB ( hai dây bằng nhau nên hai cung
bằng nhau)
 1
M
2 sđ 
AB (góc nội tiếp chắn 
1
Ta có: AB )
 1
ABC 
2 sđ 
AC (góc nội tiếp chắn AC )
M   ABC
1
 
Do tứ giác ABCM nội tiếp nên M 2  ABC
M  M 
1 2


 tia MA là tia phân giác BMx .

b) MD // CH:

MC  MH (gt)  MCH cân tại M có BMC 
 2 MHC (góc ngoài tại đỉnh M của MCH )

93
Nhóm file Word toán THCS
Ta có AB  AC (gt); OB  OC  R  AD là trung trực của BC
  DC
 DB 


 MD là tia phân giác BMC 
 BMC 
 2 BMD

 
Vậy MHC  BMD , mà hai góc ở vị trí đồng vị nên MD / / CH .

c) Tâm đường tròn đi qia bốn điểm A, I , C , K



MA là tia phân giác BMx 
nên tia đối của tia MA là tia phân giác CMH

 MA đi qua điểm K . Thao tính chất của tam giác cân có 


AIC  
AKC  90

 Tứ giác AICK nội tiếp đường tròn đường kính AC có tâm G là trung điểm AC .

d) E chuyển động trên đường nào?


 
Khi M di chuyển trên AC ta luôn có BEO  90 , mà B, O cố định nên E chuyển động trên đường tròn đường
kính OB .

Giới hạn: E  II ' của đường tròn đường kính OB ( I , I ' lần lượt là trung điểm của BC , AB ).

 
Bài 60. Cho tam giác ABC vuông tại A có ABC  60 , MA là đường phân giác của BAC . Vẽ đường thẳng
qua M và vuông góc với đường thẳng BC cắt đoạn AC tại N , cắt đường thẳng AB tại P . Gọi O là tâm
đường tròn ngoại tiếp tam giác PBC .

a) Chứng minh tứ giác PAMC nội tiếp trong một đường tròn và suy ra tam giác PMC là tam giác vuông cân.

b) Gọi I là trung điểm của đoạn thẳng PC . Chứng minh ba điểm M , O, I thẳng hàng và MO song song với
BN .

c) Chứng minh tứ giác PNOC nội tiếp.

d) Tính diện tích tam giác PBC khi AB  3cm .

Hướng dẫn giải

94
Nhóm file Word toán THCS
a) Tứ giác PAMC nội tiếp và PMC vuông cân
 
Ta có: BAC  90 (gt)  PAC  90

Mặt khác PMC  90 (gt)  M , A cùng thuộc đường tròn
đường kính CP .
Do đó tứ giác PAMC nội tiếp trong đường tròn đường
kính PC .

 MAC 
 MPC 
(hai góc nội tiếp cùng chắn MC )


BAC

MAC   450 
Ta lại có: 2 (AM là tia phân giác BAC )

 MPC  450

Thêm vào đó PMC  90 (gt)  PMC vuông cân tại M.
b) M, O, I thẳng hàng và MO // BN
IP  IC gt   IO  PC
Do (định lý đường kính và đây cung)
IM  PC (MI là dường trung tuyến của PMC vuông cân)
=> I, O, M thẳng hàng.
BPC có PM  BC và AC  BP (gt) nên N là trực tâm của BPC
 BN  PC.
Ta lại có MI  PC (cmt)  MI // BN hay MO // BN
e) Tứ giác PNOC nội tiếp
  
Ta có: PNA  ABC  60 (cùng phụ với góc BPM )
 
PNA  PNC = 180 (hai góc kề bù)

 PNC  120
  
Mặt khác POC  2PBC =120 (góc ở tâm và góc nội tiếp cùng chắn PC )
 
Suy ra: PNC = POC  120, mà N, O là hai đỉnh liên tiếp của tứ giác
 Tứ giác PNOC nội tiếp (vì cùng thuộc một cung chứa góc)
d) SPBC
AB
cos 
ABC 
ABC vuông tại A có: BC
1
  BC = AB : cos60  3:  6 cm 
2
 AC
tgABC   AC  AB.tg60  3 3 cm 
AB
AM là đường phân giác của ABC nên:
AB MB AB AC AB  AC
   
AC MC MB MC MB  MC

95
Nhóm file Word toán THCS
AB  AC 3  3 3 1  3
 
BC 6 2

2. AB 2.3( 3  1) 6( 3  1)
MB     3( 3  1)(cm)
1 3 ( 3) 2  1 2

 BM
PBM 
PBM vuông tại M nên: cos BP

BM 1
Suy ra:
BP 

cos PBM =
3  3 1 : 2
6  3 1  (cm)
AC.BP 3 3.6 3  1  
Vậy SPHC= 2

2
9 3  
3  1 (cm2 )

Bài 61. Cho tam giác ABC nhọn, nội tiếp đường tròn (O; R). Gọi H là giao điểm của hai đường
cao BD và CE của tam giác ABC.
a) Chứng minh tứ giác BCDE nội tiếp và xác định tâm I của đường tròn này
b) Gọi F là giao điểm của AH và BC. Vẽ đường kính AOK . Chứng minh:
c) Chứng minh ba điểm H, I, K thẳng hàng

d) Giả sử BC = . Tính tổng AB.CK+AC.BK theo R.


Hướng dẫn giải:
a)BCDE nội tiếp và xác định tâm I của đường tròn này. A
 
Ta có: BEC và BDC cùng nhìn BC dưới một góc vuông nên nội
tiếp đường tròn đường kính BC có tâm I là trung điểm của BC. D
 
b) AFB  ACK
ABC có đường cao BD và CE cắt nhau tại H E H O
 0
 H là trực tâm của tam giác nên AFB  90
 0 B C
Mặt khác ACK  90 (góc nội tiếp chắn nửa đường tròn) F
 
 AFB  ACK
c)H, I, K thẳng hàng
Ta có CK//BH (cùng vuông góc với AC)

ABK  900 ( góc nội tiếp chắn nửa đường tròn)
 BK  AB

Vậy BK // CH (Cùng vuông góc với AB).


 Tứ giác BHCK là hình bình hành nên hai đường chéo BC, HK cắt nhau tại trung điểm của mỗi đường, mà I
là trung điểm của BC  I, H, K thẳng hàng.
d) AC.BK + AB.CK.

96
Nhóm file Word toán THCS
  ACK
FBA và CKA có: AFB    AKC
 900 , ABF  
(hai góc nội tiếp cùng chắn AC ).
 FBA ∽ CKA(g.g) .

BF AB
  AB.CK  BF.AK
Do đó: CK AK .
CF AC
FCA ∽ BKA  g.g     AC.BK  CF.AK
Tương tự: BK AK .
 AB.CK  AC.BK  BF.AK  CF.AK   BF  CF  AK  BC.AK
3 3
 AK 2  4R 2  3R 2
4 4
Bài 62. Cho tam giác ABC nhọn, nội tiếp đường tròn (O;R) có H là trực tâm. Tia AH cắt đường tròn (O) tại E.
Kể đường kính AOF.

a) Chứng minh BC // EF và
b)Gọi I là trung điểm của BC. Chứng minh H, I, F thẳng hang và AH = 2Oi.
c)Vẽ đường tròn tâm H bán kính HA. Đường tròn này cắt các đường thẳng AB, AC lần lượt tại D và K. Chứng
minh: .
Hướng dẫn giải
 
a) BC // EF và BAE  CAF. A

 0
Ta có: AEF  90 (Góc nội tiếp chắn nửa đường
tròn).
EF  AE mà BC  AE (gt).
O M
 BC / EF ( Cùng vuông góc với AE).
 BE  CF
 N H
( Hai cung chắn giữa hai dây song song).
  C
Do đó: BAE  CAF (hai góc nội tiếp cùng chắn hai B
cung bằng nhau). E F K

b) H , I , F thẳng hàng và AH  2OI .


 
Các góc ABF , ACF là góc nội tiếp chắn nửa đường tròn đường kính AF nên:

ABF   ACF  900  BF  AB , CF  AC . Mà CH  AB, BH  AC (do H là trực tâm của tam giác ABC )
suy ra tứ giác BHCF là hình bình hành nên hai đường chéo BC , HF cắt nhau tại trung điểm mỗi đường nên
H , I , F thẳng hàng.
97
Nhóm file Word toán THCS
AH
AHF  OI 
Ta lại có: OI là đường trung bình của tam giác 2

c) AO  DK

Gọi M , N lần lượt là giao điểm của tia BH, CH với AC, AB.
   
Ta có: BMC và BNC cùng nhìn BC dưới một góc vuông nên tứ giác BNMC nội tiếp. Suy ra ABC  AMN
(góc trong bằng góc ngoài tại đỉnh đối diện của tứ giác nội tiếp)
  
Mặt khác CAF  CBF (hai góc nội tiếp cùng chắn cung CF ) nên :
 
CAF   CBA
AMN  CBF   ABF
  900  AO  MN 1

Trong đường tròn 


H ; HA  AD
, và AK là dây và HN  AD; HM  AK  MN là đường trung bình của
ADK  MN //DK  2 

Từ (1) và (2) suy ra: AO  DK

Bài 63. Cho tam giác có ba góc nhọn nội tiếp đường tròn . Các đường cao cắt nhau tại

a) Chứng minh các tứ giác nội tiếp được trong một đường tròn.

b) Dựng hình bình hành . Chứng minh rằng thuộc đường tròn và

c) Gọi là giao điểm của và , G là giao điểm của và . Chứng minh rằng là trọng tâm tam
giác .

d) Chứng minh rằng:

BFEC , AEHF
a) Tứ giác nội tiếp

98
Nhóm file Word toán THCS
A

E K

G O
F H
M

B I C

D
 
Ta có: BFC ; BEC cùng nhìn BC dưới một góc vuông nên tứ giác BFEC nội tiệp.

AFH  
AEH  90  90  180  tứ giác AEHF nội tiếp.

D  O  
BAH 
 DAC
b) và
CH  AB  gt   BD  AB  
Do tứ giác BHCD là hình bình hành nên CH // BD , mà ABD  90

  D  O 
Lý luận tương tự ACD  90  tứ giác ABCD nội tiếp đường tròn
 
AH  BC  HBA ABC  90
  ADC
CD  AC  cmt   DAC   90   
, mà ABC  ADC (hai góc nội tiếp cùng chắn AC )
 
Vậy HAB  DAC
G ABC
c) là trọng tâm
Tứ giác BHCD là hình bình hành nên hai đường chéo HD, BC cắt nhau tại trung điểm I của mỗi đường.
GI 1
AHD  
AHD có hai đường trung tuyến AI , HO cắt nhau tại G  G là trọng tâm của tam giác AG 2 ,
mà AI là đường trung tuyến của ABC và G  AI  G cũng là trọng tâm ABC

HAO  C
 B 

d)
   
Giả sử B  C Kẻ dây BK  AD tại M  AB  AK


ACB  
ABK (hai góc nội tiếp cùng chắn hai cung bằng nhau)

   
Do B  C  ABC  ABK  Tia BK nằm giữa hai tia BA, BC

 CBK 
ABC    C
ABK  B 
99
Nhóm file Word toán THCS
  
Mà HAO  B  C (vì cùng phụ với hai góc đối đỉnh)

 HAO  C
B 

    
Tương tự B  C : HAO  C  B

HAO  C
 B 
Vậy

100

You might also like